You are on page 1of 52

CHAPTER

23

Furnishing of returns of Service Tax


23.1 Furnishing of Returns [Section 70 and rule 7] Every person liable to pay service tax shall himself assess the tax due on the services provided by him and furnish a return in Form ST-3 (in triplicate) on a half-yearly basis. 'Half year' means 1st April to 30th September and 1st October to 31st March of financial year. E-filing of service tax return: See para 23.7 23.2 Due dates of filing of returns The return has to be submitted by the 25th of the month following the particular halfyear. Even a NIL return has to be filed if the assessee has not rendered any taxable service during a particular half year. In case the due date of deposit of tax or filing of return happens to be a public holiday, the service tax can be paid or the return can be filed on the next working day immediately following the holiday. Return of Service tax has to be filed within prescribed period. If not filed, penalty is leviable under section 77. A form of show cause notice to be issued by Superintendent (Service Tax) has been prescribed. Further, a late fee has to be paid along with the filing of the return of service tax if the same is filed late (in which case there would be no further penalty levied). 23.3 Penalty for delay in furnishing return [Rule 7] Return of service tax has to be filed within the prescribed period. A late fee has to be paid alongwith the filing of return of service tax if the same is filed late. Where the return is furnished late the person liable to furnish return is liable to pay to the Central Government a penalty, on the basis of period of delay subject to maximum of `2,000. Period of delay from due date Penalty to be paid Upto 15 days `500 16 to 30 days After 30 days but upto 40 days Above 40 days `1,000 `1,000 + 100 per day in excess of 30 days `2,000

23.4 Scheme for submission of return through Service Tax Return Preparer [Section 71] Without prejudice to the provisions of section 70, the Board may, by notification in the Official Gazette, frame a Scheme for the purposes of enabling any person or class of persons to prepare and furnish a return under section 70, and authorise a Service Tax Return Preparer to act as such under the Scheme. A Service Tax Return Preparer shall assist the person or class of persons to prepare and furnish the return in such manner as may be specified in the Scheme framed under this section.

680

Systematic Approach to Service Tax

Chap. 23

"Service Tax Return Preparer" means any individual, who has been authorised to act as a Service Tax Preparer under the Scheme framed under this section as may be specified in the scheme.

The Scheme to be framed by the Board: The Scheme framed by the Board under this section may provide for the following, namely: (a) the manner in which and the period for which the Service Tax Return Preparer shall be authorised; (b) the educational and other qualifications to be possessed, and the training and other conditions required to be fulfilled, by a person to act as a Service Tax Return Preparer; (c) the code of conduct for the Service Tax Return Preparer; (d) the duties and obligations of the Service Tax Return Preparer; (e) the circumstances under which the authorisation given to a Service Tax Return Preparer may be withdrawn; (f) any other matter which is required to be, or may be, specified by the Scheme for the purposes of this section. 23.5 Contents of the service tax return The return should include inter alia, monthwise details for each of the taxable service rendered by the assessee Value of taxable service charged/billed; Value of services which are exempted with reference to the notification; Value of services which are exported; Abatement Claimed with reference to the notification; Value of taxable service realized for services already rendered; Value of taxable service realized for services yet to be rendered; Amount of service tax payable/paid; Amount of education cess payable/paid; Details of CENVAT Credit. 23.6 Enclosures to the Return The following documents should be enclosed with the return: Copies of GAR-7/TR-6 challans for the months covered in the return. Memorandum in Form ST-3A in case of a provisional assessment. List of accounts maintained in relation to service tax by the assessee should be attached with the first return. Documentary Proof for adjustment of excess service tax paid in terms of rule 6(3). Worksheet of calculation of interest in case of delayed payment of service tax. 23.7 E-filing of Returns Where an assessee has paid a total service tax of `10,00,000 or more, (including the amount paid by utilization of Cenvat Credit) in the preceding financial year, he shall have to file the return electronically. The Central Board of Excise & Customs has introduced a scheme of e-filing of service tax returns. The broad scheme is as under: 1. Assessee should have the 15 digit STP Code (based on PAN) to avail the facility of e-filing of returns. 2. The assessee should indicate his 15 digit STP code in the challans used by him for the period for which the returns are being filed. (An assessee who has not

Chap. 23

Furnishing of returns of Service Tax

681

done this may also opt for e-filing, but he will have to submit copies of Challans, evidencing payment of service tax to the concerned excise formations after indicating his 15 digit STP code on each challan). 3. The assessee should file an application to the concerned excise formation at least one month in advance before the due date of filing of the return, in Annexure-I. User 'id' and 'password' for the assessee will be communicated to him within ten days after filing the application along with the necessary technical guidance. 4. After receipt of the said details the individual service provider can download form for entering details of ST3 returns and TR6 challans (GAR-7) from the central server using internet and enter the necessary details for the concerned return period. 5. The computer generates a key number which will depend on the STP code, date of filing, value of services declared and tax paid and generates an acknowledgement giving these details which can be printed by the assessee and kept in his records as evidence of having filed the return. 6. The computer will verify the fact of payment from data obtained from Focal Point Bank. Where details as declared by the assessees are not found the assessee will be contacted. 7. The Central Board of Excise & Customs has assured all assessees opting for E-filing of returns that the department will not invoke section 77 of the Finance Act, 1994 prescribing a maximum penalty of `1,000 for non-filing of ST-3 return for delay upto one month from the due date prescribed under the rules for filing such return. It is to be clearly understood that this assurance does not extend to non-payment of tax in time or mis-declaration of the value of taxable services rendered. 8. E-filing of return is mandatory in case of assessees who are liable to pay service tax (including Cenvat Credit) of an amount exceeding `10 lakh in a year. 23.8 Multiple Services Rendered In case the assessee renders multiple categories of taxable services, he can file a single return for all the categories of taxable services. However, complete information should be presented in the return month-wise and category wise. Also, the payment of service tax has to be made in separate challans or in one challan with separate codes mentioned clearly thereon. 23.9 Revision of return [Rule 7B] An assessee may submit a revised return, in Form ST-3, in triplicate, to correct a mistake or omission, within a period of 90 days from the date of submission of the return under rule 7. Where an assessee submits a revised return, the 'relevant date' for the purpose of recovery of service tax, if any, under section 73 of the Act shall be the date of submission of such revised return. If the original return has been filed late, this period of 90 days will be applicable from the date when return has been actually filed and not from the last date when such return was required to be filed with the department. MULTIPLE CHOICE QUESTIONS
1 Return of service tax has to be filed: (A) monthly

682

Systematic Approach to Service Tax

Chap. 23

(B) quarterly (C) half-yearly (D) yearly 2 Due date of filing return is: (A) 5th of the month following the particular half-year (B) 15th of the month following the particular half-year (C) 25th of the month following the particular half-year (D) 30 days of the month following the particular half-year 3 Return of service tax is to be filed in: (A) Form ST-1 (B) Form ST-3 (C) Form ST-2 4 State whether True or False (A) There is no need to file a nil return (B) Separate returns will have to be filed if multiple services are provided (C) If due date of filing return happens to be a holiday, the return can be filed on the next working day immediately following the holiday Answers to Multiple Choice questions 1. (C) 3. (B) 4. (B) (F) 2. (C) 4. (A) (F) 4. (C) (T)

Annexure Relevant Taxable Services


Announcement made by the ICAI on 18.11.2010 Applicability of services for May and November 2011 examinations: Professional Competence Examination It is clarified that in Part II: Service tax and VAT of Paper 5 : Taxation, students will not be tested on specific questions covering individual taxable services. Integrated Professional Competence Examination It is clarified that in Part II: Service tax and VAT of Paper 4: Taxation, students will be examined only in respect of the following eight taxable services:

1. Commercial Training or Coaching


Provisions at a Glance 1. Date of applicability: W.e.f. 1.7.2003, Notification 7/2003-ST, dated 20.6.2003. 2. Service defined under section 65(26): "Commercial training or coaching" means: any training or coaching provided by a commercial training or coaching centre. 3. Service provider defined under section 65(27): "Commercial training or coaching centre" means: any institute or establishment providing commercial training or coaching for imparting skill or knowledge or lessons on any subject or field other than the sports with or without issuance of a certificate and includes coaching or tutorial classes but does not include

Chap. 23

Furnishing of returns of Service Tax

683

pre-school coaching and training centre or any institute or establishment which issues any certificate or diploma or degree or any educational qualification recognised by law for the time being in force. 4. Taxable service defined: Taxable service means any service provided or to be provided: to any person, by a commercial training or coaching centre, in relation to commercial training or coaching. 5. Scope of coverage of the service: From the abovementioned definition, the scope of coverage of the service provider can be analysed as under: (i) It must be any institute or establishment: The requirement of a place of business is important for the tax incidence to occur. Accordingly, it has been clarified that tuition teachers who conduct home tuitions at the residence of the students would not be liable for service tax. However, bureau services will be covered and be liable for service tax. (ii) It should provide commercial training or coaching. (iii) The training or coaching must be for imparting skill or knowledge or lessons on any subject or field other than the sports: Thus any training or coaching relating to the field of sports is excluded. (iv) It includes coaching or tutorial classes: All sorts of coaching classes including intensive batches and tutorial batches are included. Also, postal coaching is included within the purview of service tax. (v) It does not include preschool coaching or training centre. (vi) It does not include any institute or establishment which issues any certificate or diploma or degree or any educational qualification recognised by law for the time being in force. Thus, all universities and recognized colleges would not be liable for payment of service tax. However, this exemption applies only to cases, where charges for such services are paid directly to the institutes and not to the coaching centres by the persons (students) undergoing such course. (vii) Exemption has also been provided to following institutions: (a) Vocational Training Institute: "Vocational training institute" means an Industrial Training Institute or an Industrial Training Centre affiliated to the National Council for Vocational Training, offering courses in designated trades as notified under the Apprentices Act, 1961; (b) Services provided in relation to Modular Employable Skill courses approved by the National Council of Vocational Training, by a Vocational Training Provider registered under the Skill Development Initiative Scheme with the Directorate General of Employment and Training, Ministry of Labour and Employment, Government of India. (c) Recreational Training Institute: "Recreational training institute" means a commercial training or coaching centre which provides coaching or training relating to recreational activities such as dance, singing, martial arts, hobbies. (viii) If employer provides free training to their employees, it is not taxable. However, if it hires some outside commercial coaching or training centre for imparting training to employees, then the payment made by employer to such coaching centre will be chargeable to service tax.
(1) It has been clarified for the removal of doubts that the expression "commercial training or coaching centre" shall include any centre or institute, by whatever name called, where

684

Systematic Approach to Service Tax

Chap. 23

(2)

(3)

training or coaching is imparted for consideration, whether or not such centre or institute is registered as a trust or a society or similar other organization under any law for the time being in force and carrying on its activity with or without profit motive and the expression "commercial training or coaching" shall be construed accordingly. In case of commercial training and coaching institutes, deduction will be available only to the sale value of standard text books, which are priced. Any study material or written text provided by the institute will be subjected to service tax, i.e. it will not be allowable as deduction. Training services provided online shall also be liable for service tax.

2. Information Technology Software Services


Provisions at a Glance 1. Date of applicability: W.e.f. 16.05.2008, Notification 18/2008-ST dated 10.5.2008. 2. Taxable Service defined u/s 65(105): Taxable Service means any service provided or to be provided: to any person, by any other person in relation to information technology software. Further, information technology software shall include: (i) development of information technology software, (ii) study, analysis, design and programming of information technology software, (iii) adaptation, upgradation, enhancement, implementation and other similar services related to information technology software, (iv) providing advice, consultancy and assistance on matters related to information technology software, including conducting feasibility studies on implementation of a system, specifications for a database design, guidance and assistance during the startup phase of a new system, specifications to secure a database, advice on proprietary information technology software, (v) providing the right to use information technology software for commercial exploitation including right to reproduce, distribute and sell information technology software and right to use software components for the creation of and inclusion in other information technology software products, (vi) providing the right to use information technology software supplied electronically. 3. Meaning of information technology software: "Information technology software" means any representation of instructions, data, sound or image, including source code and object code, recorded in a machine readable form, and capable of being manipulated or providing interactivity to a user, by means of a computer or an automatic data processing machine or any other device or equipment. From the above definition it would be clear that the definition of information technology software is very wide. It can include all representations of instructions. The software may be machine readable or readable using some other machine. It may be packaged or customized.
1. IT Software service provided shall be liable to service tax in all cases whether or not such service is used in the course or furtherance of business or commerce. 2. Packaged I.T. software, pre-packed in retail packages for single use, is being exempted from service tax leviable under IT Software Service, subject to specified conditions, e.g. document providing the right to use such software, by whatever name called, if any, is packed along with the software. These conditions include that either the customs duty (in case of import) or excise duty (in case of domestic production) has been paid on the entire amount received from the buyer.

Chap. 23

Furnishing of returns of Service Tax

685

3. Legal Consultancy Services


Provisions at a Glance 1. Date of applicability: W.e.f. 1.9.2009, Notification No. 26/2009-ST, dated 19.8.2009.

686

Systematic Approach to Service Tax

Chap. 23

2. Taxable Service defined u/s 65(105): Taxable Service means any service provided or to be provided: to a business entity, by any other business entity, in relation to advice, consultancy or technical assistance in any branch of law, in any manner. However, any service provided by way of appearance before any court, tribunal or authority shall not amount to taxable service. The tax is not a tax on the profession of lawyers but on the services rendered by such lawyers in the nature of advice, consultancy or technical assistance in the branch of any law. Further, the intention of the government is not to tax small and retail transactions. Accordingly, the following transactions are not taxable: Individual lawyers providing any services Services provided by any business entity to individuals The term business entity has been specifically defined through an Explanation which is as under: Explanation.For the purposes of this sub-clause, business entity includes an association of persons, body of individuals, company or firm, but does not include an individual Service Provider Service Recipient Whether taxable? Business Entity Business Entity Yes Business Entity Individual No Individual Business Entity No Individual Individual No Other than Business Entity (For example, Business Entity No charitable associations, Governments, etc.) 3. The service should be provided in relation to advice, consultancy or technical assistance in any branch of law: The services rendered should be related to advice, consultancy or technical assistance in any branch of law. Further, from the definition itself, the services which are in the nature of appearance before any court, tribunal or authority shall not be considered as taxable service since such representation is made to fight for any rights awarded by the constitution. Further, services provided in individual capacity as well as services provided to individuals shall not be taxable. Also, if the services are rendered in employment (whether full-time or part-time) the same shall not be liable for service tax. For example, many lawyers also offer their services as part-time lecturers to colleges and universities. Such services are rendered by them in their individual capacity and hence cannot be considered as taxable. 4. The service may be provided in any manner: The term "in any manner" extends the scope of taxability to cover situations where-in the services may be provided indirectly (may be through the communication mediums or through third parties). Thus if a legal consultant sub-contracts a part of the service to another legal consultant, he would still be liable for the service tax on the entire amount and not only on services which are directly provided by him.

4. Cargo Handling Services


Provisions at a Glance 1. Date of applicability: W.e.f. 16.8.2002, Notification No. 18/2002-ST, dated 1.8.2002. 2. Meaning of cargo handling services: "Cargo handling service" means loading, unloading, packing or unpacking of cargo and includes,

Chap. 23

Furnishing of returns of Service Tax

687

(a) cargo handling services provided for freight in special containers or for non-containerised freight, services provided by a container freight terminal or any other freight terminal, for all modes of transport, and cargo handling service incidental to freight; and (b) service of packing together with transportation of cargo or goods, with or without one or more of other services like loading, unloading, unpacking, but does not include, (i) handling of export cargo or (ii) passenger baggage or (iii) mere transportation of goods. 3. Taxable Service defined: Taxable service means any service provided or to be provided to any person, by a cargo handling agency in relation to cargo handling services. Analysis of definition of Cargo Handling Agency and Service: 1. The existence of the word "agency" in the definition would imply that the person handling the cargo should not be the owner of the cargo but should handle the same in a representative capacity as an agent. In case the person is the owner of the cargo, it would not amount to a service at all. 2. Based on the clarifications issued by the Department, it is felt that where individuals undertake the activity of loading or unloading of cargo (like if someone hires labour/labourer for loading or unloading of goods in their individual capacity), they will not come under the purview of service tax as a cargo handling agency. However, the same position will not prevail in case of proprietary concerns. 3. For the purpose of attracting the levy, it is important that the goods being handled should be "cargo" which means the handling of the goods while in the process of transportation. 4. The following service provided shall not be liable for service tax: (a) handing services in relation to passengers baggage. (b) handling service provided for export cargo. (c) mere activity of transportation of goods. (d) mere packing of goods. Further, taxable services provided to any person by a Cargo Handling Agency is relation to agricultural produce or goods intended to be stored in a cold storage shall not be liable for service tax. 5. Although mere packing of goods is not covered under Cargo Handling service but services of packaging together with transportation of cargo or goods with or without one or more other services like loading unloading, unpacking, etc. shall be liable for service tax. 6. If a person offers a package rate which includes transportation and handling both, unless cost of transportation is shown separately in the bill, the whole value will be 'includible' as taxable service. However, if the amount of transportation is shown separately in the invoice, service tax is attracted only on the cargo handling amount.
In a case where the role of the agency is merely restricted to provision of machinery on a hire basis and there is no performance of work, the transaction cannot be taxed under this category. However, if the activity of handling is also included, there would be a liability to service tax.

688

Systematic Approach to Service Tax

Chap. 23

5. Customs House Agent's Services


Provisions at a Glance 1. Date of applicability: W.e.f. 15.6.1997, Notification No. 17/97-ST, dated 6.6.1997. 2. Meaning of Customs House Agent: "Custom house agent" means a person licensed, temporarily or otherwise, under the regulations made under section 146(2) of the Customs Act, 1962. 3. Taxable service defined: Taxable service means any service provided or to be provided to any person, by a custom house agent in relation to the entry or departure of conveyances or the import or export of goods; 4. Analysis of definition of Custom House Agent: (1) As per Customs Act, 1962 no person is entitled to act as a custom house agent unless he is issued a licence under section 146(2) of the said Act. The issue of a licence is governed by the Customs House Agents Licensing Regulations, 1984. A custom house agent is either granted a permanent licence (renewable every 5 years) or a temporary licence (for 1 year). Both such licence holders shall be liable for payment of service tax. (2) Only licensed customs house agents are liable for payment of service tax and accordingly, unlicensed agents are outside the purview of taxation. Since Customs Act, 1962 prohibits the business of custom house agent in case there is no licence, such unlicensed agents work under the banner of a licensed agent and accordingly, the principal licensed agent will be liable for service tax for such transactions even though the transactions may be undertaken by another person. 5. The service should be the service should be in relation to the entry or departure of conveyances or the import or export of goods The services rendered by the custom house agent are not merely limited to the clearing of the import and export consignment. The CHA also renders the service of (i) loading/unloading of import or export goods from/at the premises of the exporter/importer (ii) the packing, weighment, measurement of the export goods, the transportation of the export goods to the customs station or the import goods from the customs station to the importer's premises, (iii) carrying out of various statutory and other formalities such as payment of expenses on account of octroi, destuffing, drawback survey, dock' fees, repairing and examination charges, landing and container charges, statutory labour charges, testing fees, etc. 6. Valuation of Taxable Services The value of the taxable service shall be the gross amount charged to the client (including amounts claimed as reimbursements).

6. Practising Chartered Accountant's Services


Provisions at a Glance 1. Date of applicability: W.e.f. 16.10.1998, Notification No. 53/98-ST, dated 7.10.1998. 2. Meaning of Practising Chartered Accountants: "Practising chartered accountant" means a person who is a member of the Institute of Chartered Accountants of India and

Chap. 23

Furnishing of returns of Service Tax

689

is holding a certificate of practice granted under the provisions of the Chartered Accountants Act, 1949, and includes any concern engaged in rendering services in the field of chartered accountancy. Thus from the definition above, it is amply clear that two types of entities are covered under the scope of levy for practicing chartered accountants: 1. Any person who is a member of the Institute of Chartered Accountants of India and is holding a certificate of practice granted under the provisions of the Chartered Accountants Act, 1949. 2. Any concern engaged in rendering services in the field of chartered accountancy. 3. Taxable service defined: Taxable service means any service provided or to be provided to any person, by a practising chartered accountant in his professional capacity, in any manner; It may be noted that the tax is not a tax on the profession of chartered accountancy but on the services rendered by such a chartered accountant who is in practice. Therefore two important criteria for taxability under this head would be the holding of a certificate of practice and also the rendering of services in a professional capacity of a chartered accountant. The profession of chartered accountancy is governed by the Chartered Accountants Act, 1949 and the regulations made thereunder. The service should be provided in his professional capacity The services rendered by the chartered accountant should be in his professional capacity. Therefore if the services are rendered in employment (whether full-time or parttime) the same shall not be liable for service tax. For example, many chartered accountants also offer their services as part-time lecturers to colleges and universities. Such services are not rendered by them in their professional capacity and hence cannot be considered as taxable. Representation before statutory authority also covered The service relating to representing the client before any statutory authority in the course of proceedings initiated under any law for the time being in force, by way of issue of notice were exempt as per Notification No. 25/2006. This notification has been withdrawn w.e.f. 1.5.2011 as per Notification No. 32/2011, dated 25.4.2011. Hence, representation services before statutory authorities shall now be covered under service tax. The service may be provided in any manner: The term "in any manner" extends the scope of taxability to cover situations where-in the services may be provided indirectly (may be through the communication mediums or through third parties). Thus if a chartered accountant sub-contracts a part of the service to another chartered accountant, he would still be liable for the service tax on the entire amount and not only on services which are directly provided by him.

690

Systematic Approach to Service Tax

Chap. 23

Valuation: The value of the taxable service shall be the gross amount charged to the client (including amounts claimed as reimbursements).

7. Consulting Engineer's Services


Provisions at a Glance 1. Date of applicability: W.e.f. 7.7.1997, Notification No. 23/97-ST, dated 2.7.1997. 2. Meaning of Consulting Engineer: "Consulting engineer" means any professionally qualified engineer or any body corporate or any other firm who, either directly or indirectly, renders any advice, consultancy or technical assistance in any manner to any person in one or more disciplines of engineering. The word consult implies a conference of two or more persons or an impact of two or more minds in respect of a topic in order to enable them to evolve a correct, or at least a satisfactory solution. It will be evident from the above that the scope of consulting covers engaging of experienced and knowledgeable persons to find solution to problems. It involves guiding, advising, etc., but not executing the advice. 3. Taxable service defined: Taxable service means any service provided or to be provided to any person, by a consulting engineer in relation to advice, consultancy or technical assistance in any manner in one or more disciplines of engineering including the discipline of computer hardware engineering. Explanation.For the purposes of this sub-clause, it is hereby declared that services provided by a consulting engineer in relation to advice, consultancy or technical assistance in the disciplines of both computer hardware engineering and computer software engineering shall also be classifiable under this sub-clause.
Service tax is leviable on advice, consultancy or technical assistance provided by consulting engineers.

Whether self employed professionally qualified engineers can be considered as consulting Engineers: Consulting engineers include self-employed professionally qualified engineer, whether or not employing others for assistance. Services provided by such self-employed professionally qualified engineer to a client in relation to one or more discipline of engineering is leviable to service tax under consulting engineer service [section 65(105)(g)]. Non-engineers not covered: Where the service provider, a draughtsman, was not even a graduate but was only a certificate holder from an Industrial Training Institute run by the Ministry of Labour and Rehabilitation, the Tribunal held that the service provider could not be called an engineer at all, let alone a consulting engineer, and that, in order to be a consulting engineer, one has to be professionally qualified by obtaining a degree or diploma in engineering from a recognized university. Consultancy service liable: Service tax under this category is leviable on the consultation element of the transaction and not the execution of a work. The services which attract the levy include all the services which are rendered in the capacity of a professional person and specifically include the services pertaining to structural engineering works, civil/mechanical/electrical engineering works or relating to construction management. In cases where the role of the engineering firm is merely to provide consultancy, the services will be squarely covered under this category and become liable for payment of service tax

Chap. 23

Furnishing of returns of Service Tax

691

The definition of taxable service is also very wide and shall cover services of any kind. In a particular case, the valuation of immoveable property was also considered to be within the ambit of taxable services provided by a consulting engineer services since all the advice given by an engineer based on his engineering knowledge and skill would be in the realm of engineering advice.

8. Manpower Recruitment or Supply Agency's Services


Provisions at a Glance 1. Date of applicability: W.e.f. 7.7.1997, Notification No. 23/97-ST, dated 2.7.1997. 2. Meaning of Manpower Recruitment or Supply Agency: "Manpower recruitment or supply agency" means any person engaged in providing any service, directly or indirectly, in any manner for recruitment or supply of manpower, temporarily or otherwise, to any other person. 3. Taxable service defined: Taxable service means any service provided or to be provided to any person, by a manpower recruitment or supply agency in relation to the recruitment or supply of manpower, temporarily or otherwise, in any manner. In order to provide more clarity to the scope of taxability the following explanation has been inserted. For the removal of doubts, it is hereby declared that for the purposes of this subclause, recruitment or supply of manpower includes services in relation to prerecruitment screening, verification of the credentials and antecedents of the candidate and authenticity of documents submitted by the candidate. It is not only the manpower recruitment activities, but the man power supply activities would also be liable for service tax. Manpower, recruitment or supply agency need not be a commercial concern: A manpower recruitment or supply agency is defined to mean any person engaged in providing service for recruitment or supply of manpower. There is no longer a requirement for the agency to be a commercial concern. Thus if an educational institution charges for placements, they would also be liable for service tax even though they would not be commercial concerns. This aspect has been clarified by the Board in the context of campus interviews of institutes like, IIT's, IIMs, etc. Service tax is leviable on services provided by such institutions in relation to campus recruitment. Temporary supply of manpower also covered: Where, business or industrial organisations engage services of manpower recruitment or supply agencies for temporary supply of manpower which is engaged for a specified period or for completion of particular projects or tasks, such services will be covered within the scope of the definition of the taxable service and, since they act as supply agency, they fall within the definition of manpower recruitment or supply agency and are liable to service tax. Service provided to Goods Transport Agency exempt from Service Tax: Service provided by Manpower Recruitment or Supply Agency in relation to the recruitment or supply of manpower, temporary or otherwise, to a goods transport agency for use by the said goods agency to provide any service shall be exempt from the whole of service tax leviable. Valuation of Taxable Service: The value of the taxable service shall be the gross amount charged to the client (including amounts claimed as reimbursements).

692

Systematic Approach to Service Tax PRACTICAL QUESTIONS*

Chap. 23

1. A partnership firm, gives the following particulars relating to the services provided to various clients by them for the half-year ended on 30.9.2010: (i) Total bills raised for `8,75,000 out of which bill for `75,000 was raised on an approved International Organisation and payments of bills for `1,00,000 were not, received till 30.9.2010. (ii) Amount of `50,000 was received as an advance from XYZ Ltd. on 25.9.2010 to whom the services were to be provided in October, 2010. You are required to work out the: (a) taxable value of services (b) amount of service tax payable. Ans.: `7,50,000, `77,450. [See problem 27.1 of Practical Approach] 2. Ajay Ltd. has agreed to render services to Mr. Guru. The following are the chronological events: Contract for services entered into on 31.8.2010 ` Advance received in September, 2010 towards all services 60,000 Total value of services, billed in February, 2011 2,10,000 Above includes non-taxable services of 70,000 Balance amount is received in March, 2011 When does the liability to pay service tax arise and for what amount? Contract contains clear details of services; consideration and service tax are charged separately, as mutually agreed upon. Ans.: Service tax on advance `4,120, due date 5.10.2010, on balance portion `10,300 in March 2011. [See problem 27.2 of Practical Approach] 3. R Ltd. has entered into a contract with X Ltd. on 31.7.2010 for rendering both taxable and non-taxable services. The contract provides that consideration and service tax are to be charged separately. The following information is available to you in connection with above contract: 1. Advance received on 5.8.2010 from X Ltd. towards all services `6,00,000 (exclusive of service tax) 2. Total value of services billed in August and September 2010 `21,00,000 which includes `7,00,000 for non-taxable services. Balance consideration is received in March 2011. Determine the service tax payable and the due date of payment of service tax. Ans.: 5.9.2010 `41,200, 31.3.2011 `1,03,000. [See problem 27.3 of Practical Approach] 4. The following information is furnished to you for the half year ended 30.9.2010. Total value of bills raised (exclusive of service tax) `35,00,000 which includes a bill of `3,00,000 for service rendered to a diplomatic mission and `2,00,000 for service rendered in Special Economic Zone. `15,00,000 (including `3,00,000) for services rendered to diplomatic mission was received in August 2010 whereas the services were rendered in July 2010. `16,00,000 (including `2,00,000 for services rendered in a special economic zone) was received in September 2010 `4,00,000 was received in September 2010 for which services will be provided in October. (Service tax was separately charged for such advance amount when the bill for it was raised with in 14 days) (a) Compute the value of taxable service (b) the service tax payable, and (c) due date of payment of service tax. Ans.: 5.9.2010 `1,23,600, 5.10.2010 `1,85,400. [See problem 27.4 of Practical Approach] * For solutions to these Practical Questions, refer Bharat's Practical Approach to Income Tax, Wealth Tax, CST, Service Tax & VAT (Problems and Solutions), 2011 edition, by the same authors.

Chap. 23

Furnishing of returns of Service Tax

693

5. R Ltd. is engaged in the business of providing the information technology software service and sale of packaged I.T. Software. From the following information submitted to you, compute (a) value of taxable service (b) the service tax payable (c) due date of payment of service tax for the half year ended 31.3.2011. Assume that Service Tax, if applicable, is charged extra in the bills raised. ` 05.11.2010 Bills issued to S Ltd. for providing IT Software Service 5,00,000 06.12.2010 Bill issued for packaged I.T. software which includes excise duty @ 10.3% 2,20,600 16.01.2011 Bill issued for providing IT Software Service 4,00,000 25.01.2011 Amount received against bill raised on 5.11.2010 3,30,900 09.02.2011 Amount received against bill raised for package I.T. Software 1,80,000 28.03.2011 Amount received against bill dated 16.01.2011 2,42,660 31.03.2011 Advance received for IT Software service to be rendered in April 2011 1,00,000 Ans.: 5.2.2011 `30,900, 31.3.2011 `32,960. [See problem 27.5 of Practical Approach] 6. RSG a firm of advocates submits you the following information for the half year ended 30.9.2010. Compute the (1) Value of taxable services (2) Service tax payable (3) the due date of payment of service tax. ` 5.4.2010 Bill No. 101 for consultancy service rendered to X Ltd. 3,00,000 6.5.2010 Bill No. 102 for consultancy service rendered to 'Y' 1,50,000 18.5.2010 Amount received from X Ltd. 3,00,000 6.6.2010 Bill No. 103 raised in the name of P Ltd. for appearance in Court relating to its case 2,10,000 9.6.2010 Amount of fee received for 'Y' 1,00,000 9.7.2010 Bill No. 105 for consultancy service rendered to Aparna Charitable Trust 2,40,000 18.8.2010 Amount received from P Ltd. 1,40,000 18.9.2010 Amount received for S.L.G. Ltd. for consultancy service to be rendered in October 2010 1,80,000 Bill raised are inclusive of service tax if applicable Ans.: Value of taxable services `4,35,178, Service tax payable `44,822, Due dates 5.6.2010 `28,014, 5.10.2010 `16,808. [See problem 27.6 of Practical Approach] 7. G, a Chartered Accountant, is holding a certificate of practice and has provided the following services to its clients in the half year ended on 31.3.2011. Compute the service tax payable and the due date of payment. The bills raised are inclusive of Service Tax, if applicable. ` 5.12.2010 Bill No. 101 for statutory audit fee charged to R Ltd. 3,00,000 6.1.2011 Bill No. 102 Internal Audit fee charged to S Ltd. 2,40,000 6.2.2011 Amount received for R Ltd. 3,00,000 8.2.2011 Bill No. 103 raised in the name of P Ltd. for appearance before 2,60,000 Appellate Tribunal against notice issued to the client. The cheque has been received on the same date 5.3.2011 Amount received for S Ltd. 1,60,000 8.3.2011 Amount received as part time lecturer 80,000 28.3.2011 Amount received from P Ltd. 2,60,000 Ans.: 5.3.2010 `28,014, 31.3.2011 `14,941. [See problem 27.7 of Practical Approach]

CHAPTER

24

Evaluation of VAT in India and its Justification


24.1 Historical Background Value Added Tax or VAT is a broad based tax levied at multiple stage with tax on inputs credited against taxes on output. VAT was first introduced in France in 1954. With this imposition, France became the first European country to implement VAT on an extensive scale. VAT was introduced in Brazil in Mid 1960's then in many European countries in 1970s and subsequently it has been introduced in about 130 countries. In India, most of the States including Union Territories have implemented VAT w.e.f. 1st April, 2005 which has substituted local sales tax laws and various other local levies. It will not be out of the context to mention that VAT had already been introduced under the Central Excise Law way back in 1986 and in Service Tax since 2002. Inter sectoral credit is also allowed since 10th September, 2004. Under the inter sectoral credit system the assessee is allowed to take credit of central excise duty paid on input/capital goods while making the payment of service tax. Similarly the assessee is allowed to take credit of service tax paid on input services while making payment of central excise duty. VAT in India is not comparable with that of in other countries because it is replacement of State level sales tax and not entire indirect tax structure. VAT in India is a State subject. The States Governments of Madhya Pradesh and Maharashtra had incorporated the concept of tax on value addition in their respective Sales Tax Legislations during the mid 90's, but could not succeed in implementing the same in the desired manner. With the joint efforts of the Central Government and the State Governments, VAT was implemented by a majority of the States with effect from 1st April 2005. VAT has replaced the general sales tax structure with the only difference in the manner of its levy. The power to levy tax on sales transactions in the form of VAT is drawn from entry 54 in List II of Seventh Schedule of the Constitution of India by the State Governments. Under VAT, every sale transaction taking place in the course of business is taxed enabling the State Government to collect revenue on value addition at every stage. The cascading effect of VAT, being collected at every stage on the cost of goods, is reduced by providing set off of tax paid on the purchases. 24.2 Evolution of VAT in India India already had a system of tax collection wherein the tax was collected at one point from the transactions involving the sale of goods. The single point tax was collected either at the first stage or at the last stage. The system of collecting tax at first stage had the following disadvantages: (a) Since sales tax was levied and collected at the first stage (i.e., at the stage of the wholesale), the tax rate had to be higher. This encouraged tax evasion and sales tax became a tax on honesty, which means the more the honesty, more the tax liability.

Chap. 24

Evaluation of VAT in India and its Justification

695

(b) In case somehow the goods escaped the tax at the first stage, the goods escaped tax net altogether since there was no way by which it could be caught at any subsequent stage. (c) There was ample scope for under-valuation of the value of the goods at first stage, as there was no tax payable at any subsequent stages, even if the goods were subsequently sold at much higher prices. In the system of collection of tax at the last stage also, several weaknesses were witnessed: (a) The tax evasion was maximum since the price charged at the last point of sale increases, which encouraged evasion, even if the tax rates were low; (b) It was difficult to track the goods evading tax since there was no record of their earlier movements and after the last point sale, the goods reached in the hands of the consumers; (c) This also encouraged under-invoicing and involves generation of black money due to cash dealings at the last point of sale. Since VAT is collected at various stages, all the above disadvantages and weaknesses have been overcome, the cascading effect of taxes is eliminated. More transparent structure is made up and compliance are improved. India has been slow in adoption of VAT. In domestic trade taxes, it adopted excise duty and service tax at the central level and sales tax at the State level for this purpose. The State Governments had been indifferent in undertaking any reforms in their sales tax system, although it accounts for approximately 60% of the State's own tax revenue. The existing sales tax system of the States was confronted with many drawbacks and weaknesses. The Task Force known as Kelkar Committee observed that presently, "each State levies multiple taxes on the same item in different names or at different stages e.g. Entry Tax, Luxury Tax, etc." However, it opined that "it is necessary that State VAT should be the tax to unify all the State-level taxes i.e. Sales Tax, Purchase Tax, Turnover tax, Works Contract Tax, Entry Tax, Special Additional Tax, etc. should all be covered under State VAT. The efforts were initiated towards introduction of VAT since last many years. The Committees of States' Finance Ministers (in 1995 and 1998, respectively) and of the Chief Ministers (in 1999) have put forth recommendations to replace sales tax by VAT. This was ratified by the Conference of the Chief Ministers and Finance Ministers held on November 16, 1999 and introduction of State VAT in lieu of Sales Tax was finally scheduled to be made with effect from 1.4.2003. However, the schedule had to be revised in view of agitative traders' community. The Empowered Committee of State Finance Ministers agreed upon 1.6.2003 as the revised date of implementation of VAT and it was expected that most of the State and Union Territories will implement VAT from 1.6.2003, but it did not happen. Later, the Finance Minister deferred the implementation of VAT for some more time so that more conducive environment may be created and agitative opposition may be set to peace. Besides consensus of all the States over the model law and introduction of VAT on uniform basis was also necessary. On 30.4.2003 he announced that unless all States conform to model draft law and agreed VAT rates, introducing VAT on 1.6.2003 will not be possible. He stated that VAT should be implemented all over India. Patchwork will

696

Systematic Approach to VAT

Chap. 24

not serve the purpose. In this connection, the Empowered Committee of State Finance Ministers met regularly and brought out a White Paper on State level VAT on 17.1.2005. This White Paper on State-level Value Added Tax (VAT) was presented in three parts: Part 1: In this part, the justification of VAT and its background had been mentioned. Part 2: The main design of State level VAT. While doing so, it recognized that this VAT is a State subject and therefore the States will have freedom for appropriate variations consistent with the basic design as agreed upon by the Empowered Committee. Part 3: Part 3 discussed the other related issues for effective implementation of VAT. Justification of VAT and Background In the existing sales tax structure, there are problems of double taxation of commodities and multiplicity of taxes, resulting in a cascading tax burden. For instance, in the existing structure, before a commodity is produced, goods are first taxed, and then after the commodity is produced with input tax load, output is taxed again. This causes an unfair double taxation with cascading effects. In the prevailing sales tax structure, several States levying multiplicity of taxes, such as turnover tax, surcharge on sales tax, additional surcharge, etc. With introduction of VAT, these other taxes have been abolished. In addition, Central sales tax is also to be phased out. As a result, overall tax burden will be rationalized, and prices in general fall. In the VAT, a set-off is given for input tax. VAT has replaced the existing system of inspection by a system of built-in selfassessment by the dealers and auditing. The tax structure will become simple. That will improve tax compliance and also augment revenue growth. The VAT will therefore help common people, traders, industrialist and also the Government. It is indeed a move towards more efficiency, equal competition and fairness in the taxation system. The white paper also specified that registration under the State VAT Act shall not be compulsory for small dealers with gross annual turnover not exceeding `5 lakhs. However, the Empowered Committee of State Finance Ministers has subsequently allowed the States to increase the threshold limit for small dealers to `10 lakhs.

CHAPTER

25

Design of State Level VAT


25.1 What is VAT VAT is a tax, which is charged on the 'increase in value' of goods and services at each stage of production and circulation. It is also chargeable on the value of all imported goods. It is charged by registered VAT businesses/persons/taxpayers. VAT has replaced a number of other taxes and its introduction has not resulted in either increased prices to final consumers or reduced profitability of business. VAT is levied on the difference between the sale price of the goods produced or the services rendered, and the cost thereof __ that is, the difference between the output and the input. In other words It is nothing but multi-point Sales Tax. It is collected on value addition only at each stage. Tax paid by the dealer is deducted from the tax payable collected at every point of sale and the tax already paid. 25.2 How is VAT different from the earlier Sales Tax System? 1. Earlier Sales Tax System Tax was levied at the stage of the first 1. sale or at the final stage. Thus it was levied at single stage. Successive sales (resale) of goods on 2. which tax is already paid did not attract tax VAT System Tax is levied and collected at every point of sale. Thus, it is a multi-stage tax Tax is collected at every point of sale and the tax already paid by the dealer at the time of purchase of goods will be deducted from the amount of tax paid at the next sale Dealers reselling tax-paid goods will have to collect VAT and file returns and pay VAT at every stage of sale (value addition) It is transparent and easier VAT dispenses with such forms and sets off all tax paid at the time of purchase from the amount of tax payable on sale The returns and the challans are filed together in a simple format after selfassessment done by the dealer himself At the most a few forms are required Tax on goods and services both. Self-assessment is done by dealers

2.

3.

Dealers reselling tax paid goods did 3. not collect any tax on resale and file NIL returns 4. 5.

4. 5.

Computation of tax liability is complex Sales Tax was not levied at the time of purchases against statutory forms but there was misuse of such forms resulting in tax evasion. 6. Returns and challans were filed separately and the dealers have to give numerous details 7. A large number of forms are required 8. Tax on goods only 9. Assessment was done by the department 10. Penalty for defaulters/evaders not strict

6.

7. 8. 9.

10. Penalties will be stricter

698

Systematic Approach to VAT

Chap. 25

25.3 Liability of tax under existing sales tax system and VAT if charged at each stage To curb the evasion of sales tax, the State Government had desired that instead of levying the sales tax either at first stage or last stage, it should be levied at each stage. However, levying the sale tax at each stage would have resulted into the following two anomalies under the existing sales tax system: (a) Sales tax would have been charged more than once on the same item (b) There would be sales tax on sales tax i.e. it would result into cascading effect. To overcome the above anomalies VAT was introduced so that VAT is calculated by deducting input tax credit from the tax collected during the payment period. 25.3a Input tax credit The tax paid by the dealer on its purchases of inputs and capital goods is eligible for credit while making the payment of VAT on the sale of such goods. Such credit is known as input tax credit. Such input tax credit is allowed as set off from the output tax payable by the dealers on its sale. Thus, VAT is calculated by deducting tax credit from tax collected during the payment period.
A dealer cannot claim the input tax credit if the purchases of goods and capital goods are not meant for business.

Illustration 25.1 R sells goods to S for `1,00,000. He charges sales tax/VAT @ 10% on the sale price. S sells the same goods to T by adding `50,000 as his profit and charges sales tax/VAT @ 10%. Compute the tax payable under: (a) the existing sale tax system assuming sales tax is charged at each stage (b) VAT Solution Tax liability as Sales tax Tax liability VAT Payable as per VAT Payable per existing sales tax system Value of sale made by R to S Add: 10% Sales tax ` 1,00,000 10,000 1,10,000 1,10,000 ` ` 1,00,000 10% VAT `

Sales Tax Payable 10,000

Cost of goods to S

Profit Add: 10% sales tax

50,000 1,60,000 16,000 1,76,000

Sales tax payable 16,000

10% VAT

10,000 VAT Payable 10,000 1,10,000 1,00,000 VAT credit available `10,000 50,000 1,50,000 VAT payable 15,000 `15,000 10,000 = 5,000 1,65,000

Total sales tax payable `10,000 + 16,000 = `26,000

Chap. 25

Design of State Level VAT

699

Total VAT payable `10,000 + 5,000 = `15,000 In the case of VAT, tax is payable only on value addition of `50,000 i.e. `5,000. Difference of tax: `26,000 15,000 = 11,000. The above difference of `11,000 is due to the following: (1) Sales tax has been levied @ 10% on `1,00,000 twice. Hence, the difference amount is (2) Sales tax has been charged on sales tax i.e. 10% has been charged on sales tax of Hence, the cascading effect due to extra sales tax Total difference Illustration 25.2 10% Sales tax 10% VAT Effect on Proceeds to the Effect on Proceeds to the Prices to final government Prices to final government consumer consumer A mines copper and sells it to a manufacturer B Sales tax of 10%. Total Cost to B B converts it into a wire with his labour/profit and sells it to C, a wholesaler at 100% margin Sales tax/VAT Total cost to C C sells to D, a retailer at a 20% mark-up Sales Tax/VAT Cost to D D sells it to the consumer at 100% mark-up Sales tax/VAT Cost to consumer Total Proceeds to government 100 10 110 220 10 100 10 100 200 10 `10,000.

1,000 11,000

22 242 290 29 319 638 64 702

22

20 200 240

20 10 = 10

29

24 240 480

24 20 = 4

64 125

48 528

48 24 = 24 48

The first thing we observe from the above table is that with equal tax rates of 10%, the final price to the consumer is 33% i.e. `174 (`702 `528) higher in the traditional sales-tax system. A part of the difference amounting to `77 (`125 `48) is owing to the higher tax receipts of the government. The rest of the difference, `97, is taken by higher profits of the different intermediaries B, C & D. The second thing we can observe is that almost every time the VAT is charged, it is not an expense to the person who pays it, but just an advance to the government via the supplier. This is true for all except the final customer who cannot claim the VAT deduction. Actually, he is the only one who pays the full amount. We also observe from the last two lines of the above table that the consumer is benefited by `174 (`702 `528) in the VAT system whereas the government loses by `77 (`125 `48).

25.3b Incidence of tax where more than one raw-material is used for production The incidence of tax involving more than one transaction can be explained by the

700 following illustration:

Systematic Approach to VAT

Chap. 25

Illustration 25.3 1. R is the manufacturer of two raw-materials viz., X & Y. These two raw-materials have been manufactured by taking the basic produce of mines on which Vat has not been allowed. The selling cost of raw-material X is `100 per kg. and the rate of VAT is 4% whereas selling cost of raw-material Y is `120 per kg and the rate of Vat is 12.5%. 2. S has used 1 kg each of both the above raw-materials by purchasing it from R and manufactured product Z. The quantity manufactured after allowing for loss in manufacturing is 1.8 kg. The aggregate of wages, conversion cost and profit on the sale of produce Z is `500, Thus product Z has been sold for `720 and VAT has been charged @ 12.5%. 3. T who has purchased the above product Z from S has sold the same to U for `1000 and VAT has been charged @ 12.5%. 4. U sold the product Z to the customer W for `1500 and charged VAT @ 12.5% Manufacturer S VAT charged Sale price Less: VAT credit 100 4 % 4 120 12.5% 15 Net VAT paid ` 720 @ 12.5% ` 90

19 71

Whole seller T VAT charged Sale price Less: VAT credit 720 12.5% Net VAT paid Retailer U VAT charged Sale price Less: VAT credit 1000 12.5% Net VAT paid ` 1500 @ 12.5% ` 187.5 125 62.5 ` 1000 @ 12.5% ` 125 90 35

Total VAT earned by the State Government `19 + 71 + 35 + 62.5 = 187.5

Chap. 25

Design of State Level VAT

701

25.4 Eligible purchases for availing input tax credit The input tax credit is available only when the taxable goods are purchased for the following purposes (1) For sale/resale within the State; (2) For sale in the course of inter State trade or commerce; i.e. Goods are sold to any other State or Union Territory of India; (3) To be used as (i) Containers or packing materials; (ii) Raw materials; or (iii) Consumable stores, and the goods so manufactured by the use of the above raw-materials, packing materials are sold within the State or in the course of inter State trade commerce; (4) For being used in the execution of a works contact; (5) To be used as capital goods required for the purpose of manufacture of taxable goods; (6) To be used as (a) Raw materials; (b) Capital goods; (c) Consumable stores; and (d) Packing materials/containers and goods so manufactured by the use of above items are sold in the course of export out of the territory of India.
Illustration 25.4 Will the input tax credit be available in the following cases: 1. Purchases within the State and resale of the same goods within the same State. 2. Purchases of raw-material within the State for the purpose of manufacturing a product and the sale of the manufactured product within the same State. 3. Purchase of goods from other States for the purpose of resale within the State. 4. Purchase of raw-material and consumable from other States for the purpose of manufacturing and the sale of such goods within the State. 5. Purchase of goods from within the State for the sale of same goods to other States. 6. Purchases of raw-material and consumables from within State and for the sale of manufactured goods to other States. 7. Purchase of goods from within State for the sale of same goods within the same State and other States. 8. Purchase of raw-material and consumables within State for the sale of manufactured goods within the same State or other State. 9. Purchase of goods from other States and sale of the same goods to other States. Solution 1. Yes; 2. Yes; 3. No, it is not allowed for central sales tax paid; 4. No; 5. Yes; 6. Yes; 7. Yes; 8. Yes; 9. No.

25.5 Coverage of Input Tax Credit and its Set-Off (1) Instant credit of input tax This input tax credit will be given both to the manufacturers and traders for purchase of inputs/supplies meant for both sales within the State as well as to other States, irrespective of when these will be utilized/sold. This also reduces immediate tax liability. (2) Carry forward of VAT credit: If the input VAT credit exceeds the tax payable on sales made within the State in a month, the excess credit will be adjusted against Central sales tax payable on inter State sales but if there is still excess left it will be carried over

702

Systematic Approach to VAT

Chap. 25

to the subsequent month(s) and the unadjusted VAT credit at the end of the specified period is eligible for refund.
Illustration 25.5 R purchases goods from X for `2,25,000 which includes VAT @ 12.5%. R sells 20% of goods to T in the same State by adding profit @ 25% on cost. Balance 80% of goods are sold to V who is carrying on business in another State. The profit charged in this case was 20% on cost. VAT charged is 12.5% and CST charged is 2%. Compute the input tax credit and its set off allowed to R. Solution Computation of input tax credit and its set off Purchase price of goods in the hands of R 2,25,000 100/112.5 = `2,00,000 Sale of goods to T Sale price `2,00,000 20/100 = 40,000 125/100 VAT @ 12.5% Cost to T Sale of goods to U 2,00,000 80/100 = 1,60,000 125/100 Less: CST @ 2% Input credit allowable to R `25,000 ` 50,000 6,250 VAT payable 6,250 56,250 2,00,000

4,000 CST payable `4,000 2,04,000 Input tax credit available 25,000 Less: VAT payable 6,250 18,750 Less: CST payable 4,000 Balance input credit carried forward to next month 14,750

(3) No input credit on central sales tax paid on purchases from other States There is no credit of CST if inputs are purchased from outside the State. For example if the goods are purchased by Delhi dealer from Mumbai for `1,02,000 which includes CST of `2,000, Delhi dealer will not get input tax credit of `2,000. The cost of purchase in this case shall be `1,02,000. (4) Input credit on stock transfer to other States Stock transfer to branches or on consignment basis does not amount to sale. Therefore, it is not subject to VAT or CST. However, if goods are sent outside State on stock transfer/consignment basis, credit (set off) of tax paid on inputs purchased within the State is available only to the extent of tax paid in excess of 2% as 2% is retained by the State Government. For example if tax paid on inputs is 12.5%, input credit of 10.5% is available. If the tax paid on purchase is 4%, input credit of 2% (4% 2%) is available. However, if the goods are transferred on stock transfer basis to other branches in the same State, full input tax credit shall be available to the dealer.
Illustration 25.6 R of Delhi purchased goods from X of Delhi amounting to `6,75,000 which includes VAT @ 12.5%. R sold the same goods to the following parties 50% of the Goods was sold to S of Delhi by charging profit @ 25% on cost.

Chap. 25

Design of State Level VAT

703

20% of gross was sold to T who is carrying on business in Mumbai by charging profit @ 30% on cost. Balance 30% of the Goods was transferred by R to his Branch in Hyderabad. Compute the input tax credit and its set off. Solution Computation of input tax credit and its set off Purchase price of goods in the hands of R `6,75,000 100/112.5 = 6,00,000 Less: 2% of the cost of goods on account of Input credit of 30% of the stock transferred to Hyderabad branch which is not allowed `6,00,000 30/100 = `1,80,000 2% Balance input tax credit available Sale of 50% of goods to S `6,00,000 50/100 = `3,00,000 125/100 VAT @ 12.5% `3,75,000 46,875 4,21,875 Vat payable `46,875 Input tax credit available `75,000

3,600 71,400

1,56,000 3,120 CST payable 3,120 1,59,120 Carry forward of VAT allowed to next month = `71,400 46,875 (VAT payable) 3,120 (CST payable) = `21,405

Sale of goods to U 6,00,000 20/100 = 1,20,000 130/100 CST @ 2%

(5) Treatment of input tax in case of export sales Export sales are zero rated and thereby exporters are either granted refund of input taxes paid by them or they can adjust such input tax while making the domestic sales.
Illustration 25.7 R of Delhi purchased goods from X of Delhi for `11,25,000 which includes VAT @ 12.5% R sold the goods to the following parties: 90% of the Goods was sold to S of Delhi by charging profit @ 20% on cost. 2% of the goods was sold to T of Mumbai by charging profit @ 25% on cost CST charged 2%. 2.5% of the Goods was sold to U of Germany by charging profit 10% on cost. No tax was charged. Balance 5.5% goods was transferred to the Branch at Ludhiana (Punjab). Compute the input tax credit and the set off allowed and the VAT & CST payable. Solution Computation of input tax credit and set off Cost of goods to R Input VAT credit 11,25,000 100/112.5 = 10,00,000 1,25,000 Less: 2% of cost of goods on account of input credit of 5.5% stock transferred to Ludiana (Punjab) which is not allowed 10,00,000 5.5/100 = 55,000 2% 1,100

704

Systematic Approach to VAT


Balance input credit available Sale of 90% of goods to S of Delhi Sale price 10,00,000 90% = 9,00,000 120/100 VAT @ 12.5% 10,80,000 1,35,000 12,15,000 Sale of 2% goods to T of Mumbai Sale price `10,00,000 2% = 2,00,000 125/100 CST @ 2% Sale of 2.5% goods to U of Germany Sale price 10,00,000 2.5/100 = 25,000 110/100 Tax charged 2,50,000 500 25,500

Chap. 25
1,23,900

VAT payable `1,35,000

CST payable `500

27,500 Tax payable Nil Nil 27,500 Since export sale is a zero tax rated sale, input credit of goods purchased which was sold to Germany can be set off from the VAT payable on domestic sales VAT payable on goods sold within Delhi 1,35,000 Less: Total input VAT credit available after deducting 2% for stock transferred but including input credit on purchases which have been sold to V of Germany 1,23,900 Net VAT payable 11,100 CST payable 500

25.6 Purchases not eligible for input tax credit Input credit is not be allowed in the following circumstances: 1. Purchasing from unregistered dealers; 2. Purchases from registered dealer who opt for composition scheme under the provisions of the Act; 3. Purchases of goods as may be notified by the State Government; 4. Goods where purchase invoice is not available; 5. Purchase of goods where invoice does not show the amount of tax separately; 6. Purchase of goods, which are utilized in the manufacture of exempted goods; 7. Purchase of goods used for personal use/consumption or provided free of charge as gifts; 8. Goods imported from outside the territory of India; 9. Goods purchased from other States viz. inter-State purchases; 10. Goods purchased are given away as free samples; 11. Goods purchased are destroyed by fire or are stolen or lost; 12. Goods received on consignment sale or on stock transfer from other States; 13. Goods purchased and returned within the specified period; 14. Purchase of automobile and its spare parts and accessories by a person other than a dealer. 25.7 Input tax credit on capital goods Input tax credit on capital goods will also be available for traders and manufacturers. Tax credit on capital goods may be adjusted over a maximum of 36 equal monthly

Chap. 25

Design of State Level VAT

705

instalments. The States may at their option reduce the number of instalments. There is a negative list for capital goods (on the basis of principles already decided by the Empowered Committee) which is not eligible for input tax credit. 25.8 Cenvat credit of Excise and Service Tax
Excise Duty and Service tax are levied by the Central Government. Whereas VAT is levied by the State Government. Hence, the assessee is allowed Cenvat credit of Excise and Service Tax on input goods/input services while making the payment of Central Excise Duty and Service tax under the Central Law. The Cenvat credit of Excise duty is allowed only when the buyer is also manufacturer of goods. On the other hand, input tax credit of VAT is allowed to every dealer (whether manufacturer or trader) against the VAT payable/CST payable under the State VAT Law. Central Sales Tax (CST) & Value Added Tax (VAT). 25.9 Central Sales Tax (CST) & Value Added Tax (VAT) Central Sales Tax is charged by the seller of goods when he makes an inter-State sale i.e. sale made to dealer/consumer in the other States. In this case, goods move from one State to another State. On the other hand, VAT is charged by the seller of the goods, when he makes an intra State sale i.e. sale made a to dealer or a consumer within the same State. 25.9a Who levies CST and VAT CST is levied by the Central Government but it is collected and retained by the State Government from where the movement of goods started. VAT is levied by the respective State Governments and it is collected and retained by the same State Government. Inter-State purchases of goods is not vatable:Goods which are purchased from other States are subject to CST. Such CST is not eligible for input tax credit as CST has been received by other State from where the movement of goods started. Hence, CST paid on purchases from other States is non-vatable. CST charged on the sale made to other States In case of sale made to other States, CST is charged by the selling dealer. Although such CST is levied by Central Government but it is payable to the State Government from where the Goods were sold. A selling dealer is entitled to input tax credit on the purchase made within the State for making sale of such goods to other State. Example R of Delhi sells goods to S of Mumbai (Maharashtra) and S of Mumbai sells the same goods to T of Chennai (Tamilnadu) and T of Chennai sold the same goods to U of Chennai (Tamilnadu). V of Chennai sold the same goods to consumer V of Coimbatore (Tamilnadu). In the above case, State of Delhi will get CST from R on account of sale made by him to S of Mumbai. State of Maharashtra will not allow input tax credit to S of Mumbai as tax was collected by Delhi Government. State of Maharashtra did not get any tax on purchases made from Delhi. State of Maharashtra will get CST from S on account of sale made by him to T of Chennai (Tamilnadu). State of Tamilnadu in this case will not allow input tax credit to T of Chennai (Tamilnadu) as tax was collected by Maharashtra Government. State of Tamilnadu did not get any tax on purchases made from Mumbai (Maharashtra). State of Tamilnadu will get VAT from T of Chennai (Tamilnadu) as the goods were sold within Tamilnadu. But, State of Tamilnadu will not give input tax credit to T as tax on purchases made by T from Mumbai was not received by Tamilnadu State. U of Chennai will get the input tax credit on account of goods purchased from T of Chennai as the Tamilnadu Government got VAT amount from T of Chennai. Further U of Chennai shall pay the VAT on the sale made to V of Coimbatore after setting off the input tax credit. V of Coimbatore is although in the State of Tamilnadu and has paid tax on its purchases but will not get any input tax credit as V is a consumer of goods and not the dealer of goods.

706

Systematic Approach to VAT

Chap. 25

25.9b Rate of CST If the goods are sold to a registered dealer under Central Sales Tax Act, in the other State, the rate of Central Sale Tax shall be levied at VAT rate (local sales tax i.e. LST rate) subject to maximum rate of CST which shall be 2%. In other words, CST rate shall be VAT rate or 2% whichever is less. On the other hand if the goods are sold to any other person in the other State, CST shall be equal to VAT rate of State from where the Goods are sold. It can be illustrated as under: Example VAT Rate in the State Sale to Registered Sale to any dealer in another other person in State another State 1% 1% 1% 2% 2% 2% 4% 2% 4% 12.5% 2% 12.5% Illustration 25.8 Determine how much input credit shall be available to the dealer 'X' in Delhi in respect of the following purchases: (1) Goods purchased from Mumbai `2,04,000 which includes Central Sales Tax (CST) @ 2%. (2) Goods purchased from a dealer in Delhi 'A' `3,00,000. VAT charged 12.5% i.e. `37,500. Total value of purchase invoice `3,37,500. (3) Goods purchased from unregistered dealer `24,000. (4) Goods purchased from a dealer 'Y' under composition scheme `60,000. Y has paid 1% as tax under composition scheme. (5) Purchases from dealer Z in Delhi for `1,50,000. VAT charged @ 4% `6,000. Total value of purchase `1,56,000. (6) Purchases from dealer B in Delhi `42,000 VAT is not separately charged in the invoice. (7) Purchase of capital goods `7,20,000. `6,30,000 is price of capital goods and `90,000 is VAT amount separately charged. (8) Goods purchased `62,400 which includes 4% VAT which is separately shown. Such goods have been utilized in the manufacture of exempted goods. (9) Value of goods imported from Germany `6,00,000. Solution Particulars Amount of input credit available (1) No input credit will be available on goods purchased from other States (2) Purchases from dealer 'A' in Delhi 37,500 (3) Purchase from unregistered dealer Nil (4) Purchases from dealer under composition scheme Nil (5) Purchase from dealer 'Z' 6,000 (6) Purchasers from dealer B. VAT not separately charged Nil (7) Purchases of capital goods 90,000*/36 2,500 (8) Purchases of goods used for manufacturing exempted goods Nil (9) Goods imported from Germany Nil Total VAT credit 46,000 * VAT credit on capital goods shall be allowed in 36 equal monthly instalments. Thus `90,000/36 = `2,500 VAT credit on capital goods shall be allowed every month.

Chap. 25

Design of State Level VAT

707

25.10 Coverage of Goods under VAT In general, all the goods, including declared goods will be covered under VAT and will get the benefit of input tax credit. The only few goods which will be outside VAT will be liquor, lottery tickets, petrol, diesel, aviation turbine fuel and other motor spirit since their prices are not fully market determined. These will continue to be taxed under the any other State Act or even by making special provisions in the VAT Act itself, and with uniform floor rates decided by the Empowered Committee. 25.11 VAT Rates and Classification of Commodities Under the VAT system covering about 550 goods, there will be only two basic VAT rates of 4% and 12.5%, plus a specific category of tax-exempted goods and a special VAT rate of 1% only for gold and silver ornaments. Thus there are four rates for VAT as per white paper: (i) 0% (ii) 1% (iii) 4% (lower rate) (iv) 12.5% standard rate 25.12 Non-availability of input credit in certain cases In the following cases credit of tax paid on inputs shall not be allowed: 1. Where final product is exempt Credit of tax paid on inputs is available only if tax is paid on final products. When final product is exempt from tax, credit will not be allowed. If credit was availed, it will have to be reversed on pro rata basis. 2. No credit if input lost/damaged/stolen before use Where the inputs have been lost or damaged or stolen before these have been used, credit of tax paid on such input shall not be allowed. If credit was availed, it will have to be reversed. 3. No credit on certain purchase Generally, in following cases, credit is not available (a) Purchase of automobiles (b) Fuel. However, some States are allowing input credit for the same. Goods under VAT From the above discussion, it may be summarized that Goods under VAT can be classified as Taxable goods and non taxable goods (Exempted goods). Taxable goods can be further divided into two parts: (a) VAT able goods i.e. goods for which input tax credit shall be allowed (b) Non-vatable goods i.e. goods for which input tax credit shall not be allowed: Goods

Taxable goods

Non-taxable goods (i.e. Exempted goods)

Vatable goods (input VAT credit) allowed

Non-vatable goods (input VAT credit not allowed)

708

Systematic Approach to VAT Goods on which Special rate of VAT is applicable e.g. liquor, lottery tickets petrol, diesel etc. Goods purchased from a dealer under composition scheme (See later)

Chap. 25

Exempted goods: Under exempted goods category, the empowered committee has listed about 50 commodities comprising of: (a) natural products; (b) unprocessed products; (c) items which are legally barred from taxation; and (d) items which have social implication. Further, a set of maximum 10 commodities out of commodities listed in the exempted category will be flexibly chosen by individual States which are of local importance for the individual States without having an inter State implication. Examples of few exempted categories of commodities are given below: (1) Books, periodicals and journals including maps, charts and globes (2) Blood including blood components (3) Fresh vegetables and fruits (4) Earthern Pot (5) Electricity energy (6) Course grains other than paddy, rice and wheat (7) Fresh plants, saplings and fresh flowers (8) Kum Kum, Bindi, Sindur, etc. (9) All bangles except those made of precious metels (10) Curd, Lassi, butter milk and separated milk (11) Betel leaves (12) Animal driven or manually operated agricultural implements their spare parts, components and accessories

CHAPTER

26

Principles, Variants and Methods of Computation of VAT


26.1 Origin/destination principle The following two principles are relevant for implementation of VAT (a) Origin principle. (b) Destination principle. (a) Origin principle: Under 'origin principle', value added domestically on all goods whether they are meant for exports or to be consumed in India is subjected to tax. Hence, if there is value added abroad tax cannot be levied on such value added in India. This principle confines VAT only to goods originating in the country of consumption. Whereas exports are taxable under this principle but imports are exempt. It is mostly used in conjunction with income VAT and is unpopular for obvious reasons. (b) Destination principle: Under this principle, value added irrespective of the place of origin is taxable. All goods are taxed if they are consumed within the country. Consequently, exports are exempt while imports are subjected to tax. Destination principle is normally used along with consumption VAT. In a federal set-up like India, destination principle is preferred for taxation of products consumed within the various States of the country. A very important feature of this principle is that imported goods are treated at par with domestic products whereas in the origin principle imported goods are not taxable and hence it gives preference to goods produced abroad. In the EEC countries, origin principle was once considered for eliminating border controls and problems of valuation, but was subsequently given up as being impractical. Thus the destination principle is now being followed in those countries. 26.2 Variants and Methods of Computation of VAT (1) Variants of VAT VAT has three variants namely, (a) gross product variant, (b) income variant, and (c) consumption variant. (2) Methods of computation of VAT The above variants can be further distinguished according to their methods of calculation, (i) addition method, (ii) invoice method and (iii) subtraction method. (a) Gross Product Variant In case of Gross Product Variant, tax is levied on all sales but deductions on all purchases of raw materials and components (i.e. inputs) are allowed. However, no deduction is allowed for taxes paid on capital inputs like plant and machinery, etc. for taxes with the result that in this variant of VAT, capital goods carry a heavier tax burden

710

Systematic Approach to VAT

Chap. 26

as they are taxed twice i.e. at the time of purchase of such capital goods and at the time of sale thereof. (b) Income Variant In case of Income Variant, tax is levied on all sales but deductions for taxes are allowed on the following: (1) purchases of raw materials and components (i.e. inputs) (2) tax on depreciation on capital goods Credit of tax paid on purchase of the capital goods is allowed in the ratio of depreciation over the life of the asset. The depreciation to be provided is dependent on the life of an asset as well as on the rate of inflation, therefore there are many difficulties connected with the variant in measuring depreciation. (c) Consumption Variant In case of Consumption Variant, tax is levied as all sales but deduction for taxes allowed on the following: (1) purchase of raw-materials and components (2) capital goods Thus, gross investment is deductible in calculating value added. This variant of VAT does not distinguish between capital and current expenditures hence there is no need to specify the life of assets or depreciation allowances for different assets. However, input tax credit on capital goods is being allowed by the States in 24/36 instalments. The consumption variant of VAT is most popular and widely used variant among the three variants of VAT. The reasons behind the preference of this variant over the other are as under: (a) This variant is tax neutral as it does not affect decisions regarding investment because the tax on capital goods is also set-off against the VAT liability. (b) The consumption variant is convenient from the point of administrative expediency as it simplifies tax administration. (c) It does not cause any cascading effect. Thus, VAT is payable under the above 3 variants as under: Gross product variant VAT payable on sales VAT credit allowed on inputs goods only. Income variant VAT payable on sales VAT credit allowed on input goods and proportionate VAT credit allowed on depreciation on capital goods. Consumption variant VAT payable on sales VAT credit allowed both on input goods and capital goods.
Illustration 26.1 R submits you the following information. Compute(a) the setting price of the product (b) cost to the consumer (c) VAT payable to Government under: (i) Gross product variant (ii) income variant and (c) consumption variant. Solution ` Purchases of Raw-material and component 4,00,000 4% VAT on the above purchases 16,000 Purchase of capital goods being machinery (life 10 years) 12,00,000 12.5% VAT on the above machine 1,50,000 Direct and Indirect Expenses 3,00,000

Chap. 26

Principles, Variants and Methods of Computation of VAT

711
20% on total cost 12.5%

Profit VAT payable on sales Case I: Gross product variant Computation of cost Raw-material cost Direct and Indirect expenses Depreciation on Machine 10% of `13,50,000 (VAT credit not available) Total cost + profit @ 20% of cost (i) Sale price Add: VAT 12.5% (ii) Cost to consumer VAT collected from consumer Less: VAT credit on purchases 4% on `4,00,000 (iii) VAT payable Income variant Raw-material cost Direct or Indirect Expenses Depreciation on machine 10% of `12,00,000 (VAT credit available) Total cost + 20% profit (i) Sale price + 12.5% VAT (ii) Cost to consumer VAT collected VAT credit for purchase of raw material Proportionate VAT credit allowed on depreciation 1,50,000 1,20,000/12,00,000 16,000 15,000

4,00,000 3,00,000 1,35,000 8,35,000 1,67,000 10,02,000 1,25,250 11,27,250 1,25,250 16,000 1,09,250 4,00,000 3,00,000 1,20,000 8,20,000 1,64,000 9,84,000 1,23,000 10,07,000 1,23,000

31,000 (iii) VAT payable 92,000 The balance VAT credit of `1,35,000 shall be allowed in subsequent 9 years proportionately. Consumption Variant Raw-material cost 4,00,000 Direct and Indirect Expenses 3,00,000 Depreciation on machine @ 10% of `12,00,000 (VAT credit available) Total cost Profit (i) Sale price VAT @ 12.5% (ii) Cost to consumer VAT collected Input credit on purchase 1,23,000 16,000 1,20,000 8,20,000 1,64,000 9,84,000 1,23,000 10,07,000

712
VAT credit on capital goods

Systematic Approach to VAT

Chap. 26

1,50,000 1,66,000 Balance credit available 43,000 (iii) VAT payable Nil Note.Most of the States are allowing VAT credit on capital goods in 24 to 36 monthly installments instead of the full amount at one time.

(2) Method for Computation of tax There are several methods to calculate the 'Value Added' to the goods for levy of tax. The three commonly used methods are as under: (A) Addition method (B) Invoice method/tax credit method, and (C) Substraction method. (A) Addition method In this case VAT is levied only on the value of addition made by a manufacturer or a dealer which will also include profit charged by him.
Illustration 26.2 R purchases raw material for `1,00,000 (excluding VAT of `4,000) and incurred the following manufacturing and trading expenses: (1) Direct and indirect manufacturing expenses excluding depreciation (2) Depreciation on assets used for manufacturing activities (3) Trading expenses (4) Depreciation on assets used for purposes other than manufacturing (5) Profit VAT payable Compute the tax payable by following the addition method. Solution Total additional cost incurred for the goods manufactured and sold `80,000 + 10,000 + 15,000 + 4,000 + 20,000 VAT payable @ 4% on `1,29,000 ` 80,000 10,000 15,000 4,000 20,000 4% on sales

1,29,000 5,160

(B) Invoice method/Tax credit method Tax credit method involves payment of tax by the seller i.e. manufacturer or dealer at full selling price and credit of tax is allowed, which he has paid at the time of purchase. Thus, the tax is levied on full sale price, but credit is given of tax paid on purchases and effectively, tax is levied only on 'Value Added' only. It's an easy and simple way to ensure that tax is paid. It helps elimination of cascading effect of tax on consumers.
Illustration 26.3 R a 'manufacturer' sells goods in Delhi to 'S' a distributor for `10,000. R was not entitled to VAT credit on the purchases of raw material as the raw-material was not liable for VAT. S, the distributor sells the same goods to whole sale dealer T for `12,000 (which includes freight and other expenses `1,500 and his profit `500). T sells the same goods to dealer U for `13,000 and V sold the same goods to consumer V for `15,000. Compute VAT payable at each stage assuming rate of VAT at each stage is 12.5%. Solution ` Sale by R to S ` `

Chap. 26
Sale price VAT @ 12.5%

Principles, Variants and Methods of Computation of VAT


10,000 1,250 12,500 12,000 1,500 13,500 13,000 1,625 14,625 Input VAT credit available Nil

713
VAT payable 1,250

Sale by S to T Sale price VAT @ 12.5%

1,250

1,500 1,250 = 250

Sale by T to V Sale price VAT 12.5%

1,500

1,625 - 1,500 = 125

Sale by U to V Sale price VAT 12.5%

15,000 1,625 1,875 Cost to consumer V 16,875 Total tax payable to the Government = `1,250 + 250 + 125 + 250 = `1,875

1,875 1,625 = 250

Thus, the Government will get tax on the final retail sale price of `15,000. However, the tax will be paid in installments at different stages. At each stage, tax liability is worked out on the sale price and credit is also given on the basis of tax charged in the purchase invoice. If the first seller is a manufacturer, he will get the credit of tax paid on raw materials, etc. which are used in the manufacturing. From the above illustration, it is clear that under this method tax credit cannot be claimed unless and until the purchase invoice is produced. As a result, in a chain, if at any stage the transaction is kept out of the books, still there is no loss of revenue. The department will be in a position to recover the full tax at the next stage. Thus, the possibility of tax evasion, if not entirely ruled out, will be reduced to a minimum. However, proper measures should be implemented to prevent the production of fake invoices to claim the credit of tax at an earlier stage. It is said that in this method the beneficiary is the trade and industry because in the above example, the total tax collection at the various stages is `6,250 (`1,250 + 1,500 + 1,025 + 1,875) whereas tax received by the State is only `1,875. The set-off available is also tax paid. If the profit margin is to be kept at the Constant level then the set-off will have to be considered to avoid cascading effects of taxes.
Illustration 26.4 Manufacturer A of Mumbai sold product X to B of Delhi @ `1000 per unit. He has charged CST @ 2% on the said product and paid `80 as freight. B of Delhi sold goods to C of Delhi @ `1250 per unit and charged VAT @ 12.5%. C of Delhi sold goods to D, a consumer @ `1500 per unit and charged VAT @ 12.5% Solution B Liability of VAT ` Cost of product X purchased from Mumbai `1000 + 20 (CST) + `80 (Credit of CST shall not be allowed under VAT) 1,100

714
Sale price VAT payable C Liability of VAT Purchase price exclusive of VAT VAT credit to be taken Sale price VAT payable @ 12.5% VAT credit allowed Net VAT payable

Systematic Approach to VAT

Chap. 26
1,250 156.25 1,250 156.25 1,500 187.50 156.25 31.25

(C) Substraction method Under subtraction method, the purchase price is deducted from selling price and tax is paid on the net amount only i.e. value added. Thus, when the tax is paid on net amount, dealer's margin is disclosed. This method is unpopular and cumbersome. It is practically impossible when various inputs are used in the manufacture of numerous outputs. It is also not preferred by dealers as their margin gets disclosed. The substraction method can be divided into: (a) Direct substraction method: In this cases aggregate value of purchases exclusive of tax are deducted from the aggregate value of sale exclusive of tax. (b) Intermediate substraction method: In this case aggregate of value of purchases inclusive of taxes deducted from the aggregate value of sale inclusive of tax. Example Suppose a manufacturer sells goods to a trader for `220 which includes tax charged @ 10%. The trader sells the same goods to a consumer for `308 which also includes tax charged @ 10%. The tax in this case shall be worked as under: Direct subtraction method Sale price exclusive of tax `280 200 (Purchase price exclusive of tax) = Value addition `80 `8 8 [88 10/110]

Tax Intermediate subtract method Taxable turnover shall be 308 220 = 88

88

Tax payable `8. In the above system also, the incidence of tax is at each stage. The subtraction method of computing VAT is normally applied where: (a) the tax is not charged separately and (b) the same rate of tax is attracted on all purchases including consumables and services added at all the stages of production/distribution. 26.3 Advantages of VAT VAT being a broad based tax levied at multiple stages is generally perceived as an explicit replacement of State sales tax for raising additional revenue for the Government. The purpose of a tax system is to bring in revenues to the Government. Tax revenues can be raised in many ways. However, the main characteristic of good tax system should

Chap. 26 be

Principles, Variants and Methods of Computation of VAT

715

The tax system should be fair or equitable; It should cause the least possible harmful effects to the economy and to the extent possible, it should promote growth to the economy; It should be simple both for its compliance by the payer and for its administration by the Government; It should be income elastic. Keeping in view the above objectives, VAT is being implemented in various States in place of the local sales tax payable by the seller. VAT is also expected to be more effective and efficient for every person including Government, manufacturers, traders and consumers and hold the following advantages: (i) Easy to Administer & Transparent This system of charging tax is easy to administer because of its simplicity. It also reduces the cost of compliance by the dealers and is transparent, as tax is to be charged in every bill and there will be no local statutory forms. (ii) Less Litigation There will be no litigation with respect to allowability of items, as under VAT no items will be specified in the registration certificate of the dealer. The dealer will be allowed to purchase any of the items of his choice in which he intends to deal. He will also be allowed to purchase any item he requires as raw material for the purpose of manufacturing or for packing. (iii) Tax Credit on purchase of Capital Goods The dealer will be allowed to purchase capital goods for manufacturing after paying VAT and will be entitled to get set off tax paid on such purchases from his VAT liability, which will arise on the sales made by him. (iv) Abolition of Statutory Forms There are no forms under VAT. Therefore, all problems related to forms automatically get resolved. Dealer will not have to make visits to department to get these forms issued. Similarly, there will be saving to the department both in terms of cost and in terms of time. Assessing Officers will be saving lot of time which otherwise was being wasted in issuance of statutory forms, and now this can be utilised for other useful purposes including monitoring of tax collection and better administration. Tax collection cost of the department will go down. There will be a direct reduction and the amount spent on the cost of paper and printing of statutory forms will become NIL. Secondly, the department will also be able to serve more dealers with existing staff and that too more efficiently. (v) Self-Assessment Dealers are not required to appear before the Assessing Authority for their yearly assessments, as under VAT there is provision for self-assessment. All the cases will be accepted by the department as correct and only a few will be selected for audit as is being done by Income Tax Department and Excise Department at present. (vi) Deterrent against Tax Avoidance It will act as deterrent against tax avoidance. Under the present system, tax is charged either on first point basis or at last point basis hence the incentive to evade tax is high because the dealer saves the whole amount of tax due on such transaction, whereas under VAT the incentive to evade tax is low because the dealer saves only a part of tax

716

Systematic Approach to VAT

Chap. 26

i.e. (tax amount which he is liable to pay less the amount of tax he has already paid on his purchases). Even otherwise the chances of not seeking sales invoice by the subsequent purchaser is less (unless otherwise he is a consumer and not registered with the department) as he will not be entitled to claim tax benefit for the tax paid by him unless he is in possession of the tax invoice. (vii) No Cascading Effect It does not have cascading (tax on tax) effect due to system of deduction or credit mechanism. Since VAT does away with cascading, it avoids distorting business decisions; the need for vertical integration is dictated only by the market forces or technical considerations, and not by the tax structure. (viii) Effective Audit & Enforcement Strategies The input credit method by generating a trail of invoices is argued to be system that encourages better compliance since the purchaser seeks an invoice to get input tax credit. Further, this trail of invoices supports effective audit and enforcement strategies. (ix) Minimum Exemptions The system will be more effective because of minimum exemptions. (x) Removal of Anomaly of First Point Taxation VAT eliminates the limitations of single point tax either at first point or last point. In the case of last point goods, the temptation to evade tax is high. Firstly, the quantum of tax at one point is very high. Secondly, as the exemption is available against statutory forms, possibility of misuse of forms cannot be ruled out. Similarly, under first point tax system, tax avoidance by way of selling the goods at first point to their sister concerns at lower rates and thereafter increasing the price of the goods because subsequent sales being exempt as tax paid. This anomaly is also being taken care of under VAT, without introducing cascading. Since the dealer gets a set off for taxes paid at the earlier stages these are not treated as part of costs and this is expected to reduce that component of cost as well as the associated financing requirement. Further, the problem of enhanced cascading via the markup rule too is also ruled out under the system. (xi) Competitiveness of Exports Export can be freed from domestic trade taxes in real sense. (xii) Instrument to tax consignment of goods Instrument of taxing consignment of goods and services as in the case of consignment transfer and stock transfer, the input tax credit is given after deducting 2%. Such 2% tax is retained by the State Government. Looking at the advantages of VAT, it appears that the amount of benefits which the society will be reaping from its implementation will be enormous. 26.4 Limitations of VAT India being a Federal Republic country has State level administration of the local sales tax which is being replaced by VAT and had been the reason for deferment of its implementation time and again. Inherently there are certain limitations of VAT due to which it being opposed by some of the trade associations. Moreover VAT undoubtedly has many advantages but without taking note of the limitation of VAT, one is just looking only at one side of the coin. The limitations of VAT are discussed hereunder: (i) Detailed Records Like any other system VAT is also not free from all evils. Though on record it is said

Chap. 26

Principles, Variants and Methods of Computation of VAT

717

to be the simplest method, however, it is more complicated than a simple first point tax. Many small dealers maintain only primitive accounts and it is very difficult for them to keep proper and detailed records required for VAT purposes. It will also be difficult to administer the tax systems at wholesale and retail stage as they usually deal in numerous products and commodities, which carry different rate. Thus matching of output and input taxes is difficult. Ideally VAT should have very few rates which does not seem to be possible in India due to varying and diverse fiscal and social requirements. In case matching requirement is waved off there is a possibility of tax evasion as explained hereunder: Purchases made by a Dealer Amount (In `) 1% 4% 8% 20% Total 1,000 1,000 1,000 1,000 4,000 Tax Paid 10 40 80 200 330 Sales Shown by Dealer Case A I II III IV Total 1,200 1,200 1,200 1,200 4,800 12 48 96 240 396 Tax Liability (`396 `330) = `66 Case B I II III IV Total 1,400 1,200 1,200 1,000 4,800 14 48 96 200 358 Tax Liability (`358 `330) = `28. (ii) Causes Inflation It is also argued that VAT causes inflation. It's impact will depend on various factors such as inventory holding period, demand supply position of that particular product, number of intermediaries etc. Investment in stock is bound to increase as tax will be paid at the time of purchase, hence one will have to carry tax paid stock. (iii) Refund of Tax Credit of tax paid on inputs/capital goods is available to be utilised against tax liability which will be calculated on the sale of final product. VAT credit can not be availed if no tax is payable on final product being exempt or taxable at lower rate. Similar situation will arise in the case of exporters also. Hence a mechanism is needed for the early refund of tax paid on input/capital goods in the case of dealer/manufacturer of exempted goods or exporters. Similarly refunds will also arise on sales made to diplomats/diplomatic missions and to other organisations who are exempted from the levy of sales tax. (iv) Increase in investment Dealer will be making purchases after paying tax, therefore investment in stock will go up the extent of tax paid. Under old system the dealer was making purchases against statutory forms, hence was not liable to pay tax on it's purchases. (v) No Credit for Tax paid on Inter-State Purchases The biggest problem of introduction of VAT is the non-availability of credit for tax paid on inter State purchases in initial years. It will also result in some cascading effect, which goes against the basic spirit of VAT.

718

Systematic Approach to VAT

Chap. 26

(vi) Composition Scheme Introduction of composition scheme will obstruct the flow of audit trail and this scheme can be misutilised by unscrupulous dealer. (vii) Audit under VAT Most of the States introduced VAT on 1.4.2005 and they have incorporated audit provisions in the Legislation itself. Audit under VAT is important for better and effective implementation of the VAT system. However, it is better to go for external Audit instead of internal audit by the department. Audit may in certain cases cause undue hardship to the dealer by the authorized official of the department. 26.5 Reasons why external audit under VAT should be compulsory (A) Lack of Education among Traders Community In our country the trading community is not educated enough therefore they face problem in understanding the requirements of tax laws. Moreover the VAT system of taxation is new to the trading community. Due lack of knowledge and unawareness, the traders are not well equipped to understand the implications of the VAT system of taxation. Keeping these factors in view the State Government in order to arrange their business affairs to fall in line with the requirements of the State Level VAT, calculate and discharge their exact tax liability under the VAT Law should incorporate External audit provisions in VAT Acts. (B) Lack of Resources with Taxation Authorities The taxation authorities do not have sufficient resources to educate the tax payers and inform them about the procedural requirements and accounting changes that are required under VAT system. Due to lack of resources, the taxation authorities are also not in a position to ensure that all the requirements of VAT are being fulfilled and there is no loss of Government revenue. Therefore, it is desirable to prescribe for an audit under VAT by a qualified professional so that the taxation authorities may comply with the procedural requirements. (C) Self-Assessment under VAT regime Another reason for prescribing an audit under the VAT by a Chartered Accountant, is that under the VAT system a major thrust is to be laid on the 'self assessment'. The dealer/assessee calculates its tax liability himself and thereafter pays the same. The tax payers through their periodical returns inform the Department about its business affairs. These periodic returns are accepted by and large and the tax payers are not to be called for substantiating their tax liability as shown by them in the returns by producing books of account and other relevant material. The assessments with books of account will be an exception. Therefore there is a strong need to see that the tax payers discharge their tax liability properly while filing the returns. This can be ensured only where the particulars furnished by the tax payers are verified by an independent auditor in minute details by going not only through the books of account but also by analyzing and interpreting the provisions of the State Level VAT Laws and reporting, whether any under-assessment was made by the dealer requiring additional payment or whether there was any excess payment of tax warranting refund to the tax payer. Due to these factors and requirements audit under VAT become essential and shall be performed on a regular basis by the professionals. However, it is not possible to conduct the audit of all the VAT dealers. Therefore, the criteria for audit can be the amount of turnover or the class of dealer dealing in specified commodities. 26.6 Roll of a Chartered Accountant under VAT The Chartered Accountant has the following key role to play in proper

Chap. 26

Principles, Variants and Methods of Computation of VAT

719

implementation of VAT: (i) Record keeping: VAT requires proper record keeping and accounting. Systematic records of input credit and its proper utilization is necessary for the dealer to take input tax credit. No doubt, the Chartered Accountant is well equipped to perform these activities. (ii) Tax planning: Chartered Account is competent to analyze various alternatives and its impact on dealer so as to minimize the tax impact. (iii) Negotiations with suppliers to reduce price: VAT credit alters cost structure of goods supplied as inputs. A Chartered Accountant will ensure that the benefit of such cost reduction is passed on by the suppliers to his company. (iv) Helping to departmental officers: There will be audit wing in department and certain percentage of dealers will be taken up for audit every year on scientific basis. Chartered Accountant can ensure proper record keeping so as to satisfy the departmental auditors. (v) External audit of VAT records: Under VAT system, self assessment has been brought into force. Chartered Accountants can play a very vital role in ensuring tax compliance by audit of VAT accounts.

CHAPTER

27

General requirement for VAT System


27.1 Compulsory issue of tax invoice and Retail Invoice The entire design of VAT with input tax credit is crucially based on tax invoice, or retail invoice. Tax invoice: Every registered dealer, having turnover of sales above an amount specified, shall issue to the purchaser serially numbered tax invoice with the prescribed particulars. This tax invoice will be signed and dated by the dealer or his regular employee, showing the required particulars. Tax invoice must contain TIN of buyer as well as seller. The dealer shall keep a counterfoil or duplicate of such tax invoice duly signed and dated. Failure to comply with the above will attract penalty. The purchaser will get input credit on the basis of the said tax invoice. Tax invoice cannot be issued by a dealer under composition scheme. Tax invoice cannot be issued in case of sale in the course of inter State sales. Particulars required to be shown in the tax invoice (a) The word 'tax invoice' at a prominent place. (b) The name, address, registration number and TIN No. of the selling registered dealer. (c) The name address registration number and TIN No. of the purchasing dealer. (d) Tax invoice should be preprinted serialized number and date on which invoice is issued should be mentioned. (e) Description, quality, volume and value of goods sold and service provided and the amount of tax charged thereon indicated separately duty. (f) The signature of selling dealer or his employee or manager, etc. duty authorized by him. (g) The name and address of the printer and the first and last serial number of tax invoices printed and supplied by him to dealer. Retail invoice: Where sales are made to a consumer or in the course of inter-State trade and commerce, the dealer shall issue retail invoice. No input credit is available to purchaser on the basis of this retail invoice. The particulars to be mentioned in the retail invoice are more or less are similar except the word retail invoice should be mentioned at prominent place. 27.2 Registration There is a compulsory registration of every dealer if the aggregate turnover exceeds a certain specified limit. It is `5,00,000 in most of the States whereas in certain States Delhi it is `10 lakhs. Small dealers whose gross turnover does not exceed `5 lakh/10 lakh shall not be liable to pay VAT. These small dealers will not be allowed input tax credit on their purchases. Small dealers may also get voluntary registration and come under the purview of VAT provisions.

Chap. 27

General requirement for VAT System

721

Compulsory registration is also necessary based on certain transactions Registration

Compulsory Registration

Voluntary Registration

Based on Turnover

Based on Transaction

Turnover is less than `5,00,000 or `10,00,000 as the case may be Advantages of voluntary Registration The buyer from such

Compulsory, if Turnover of the dealer exceeds the threshold limit of `5,00,000/10,00,000. Registration has to be applied within 30 days of crossing the threshold limit of `5,00,000/ 10,00,000

Registration is compulsorily in the following cases irrespective of the turnover of the dealer:

dealer will get in put tax credit. He can take input tax credit on the 1. Dealer is purchasing purchases goods from other made by him States 2. Dealer is selling goods to other States 3. Dealer is exporting and importing goods from a country outside India 4. Dealer is liable to pay tax at special rate i.e. 20% on goods like liquor lottery tickets, petrol, diesel, etc. Cancellation of Registration: Registration can be cancelled by the VAT authorities in the following cases: (a) The dealer has discontinued the business (b) The dealer has become insolvent (c) There is a charge in the constitution of the business (d) The dealer has sold the entire business 27.3 Composition scheme The VAT Acts are designed so that high value tax payers should not be spared and on the contrary small dealers should be hassle free from compliance procedure. The object of all such composition schemes is not to burden small dealers by the

722

Systematic Approach to VAT

Chap. 27

provisions of record keeping. Therefore, such schemes will generally contain the following features: (1) Such dealer shall require registration under VAT Act of a State. (2) A small dealer whose turnover does not exceed a specified limit of `25 lakhs. (In Delhi it is `50 lakhs) can opt for composition scheme. (3) He shall have to pay tax himself at a small percentage of gross turnover. (4) Such dealer shall not be allowed input credit on the purchases made by it from other dealers. (5) A simple return form to cover longer return period shall be sufficient. (6) The buyer of goods from a dealer under composition scheme will not be entitled to input VAT Credit. (7) The dealer covered under the composition scheme shall not be required to maintain lengthy as are records required under VAT. (8) Such scheme is a optional scheme. Who is not allowed to opt for composition scheme The following dealers are not allowed to opt for composition scheme: (1) A manufacturer or any other dealer who makes an inter State purchases. (2) A manufacturer or any other dealer who makes an inter State sale. (3) A manufacturer or any other dealer who exports or import the goods to a country outside India. (4) A dealer who wants to issue VAT invoice. (5) A dealer who wants to transfer goods outside the State otherwise than by way of sales. Advantage of composition scheme The following are the advantages of composition scheme: (1) The dealer is not required to maintain the detailed VAT records. (2) The dealer is required to file return in a simple form and it is required to be filed for a longer period instead of monthly or quarterly. (3) The dealer has to pay only a small percentage of tax. However it cannot be separately collected from the purchaser but can be included in the sale value of goods. (4) It simplies tax calculation as No input tax credit is available. Disadvantages of composition scheme 1. No input tax credit can be availed by the dealer under composites scheme on its purchases. 2. No input tax credit will be available to the dealer who has purchased goods from the dealer who is under composition scheme. 3. VAT chain breaks on opting of composition scheme as the person under composition scheme cannot issue tax invoice which is main requirement of availing input tax credit and continuing of VAT chain. 27.4 Tax Payer Identification Number (TIN) There will be a taxpayers identification number of 11 digit numericals which will be unique to each dealer. First two characters will represent the State Code as used by the Union Ministry of Home affairs. The set up of the next nine characters may, however, be

Chap. 27

General requirement for VAT System

723

different in different States. TIN is useful to the department of VAT in case of computer application, cross checking of information relating to sales and purchases across the State VAT dealers. 27.5 Records to be maintained under VAT System The following records are required to be maintained under VAT: (1) Purchase records: The dealer shall have to maintain complete records of purchases made by him as he has to take input tax credit on the basis of VAT invoices. Further, the dealer shall have to keep copies of all credit and debit notes issued in chronological order. Further, he shall also have to maintain records of purchases made without payment of tax, purchase made from a composition dealer and purchases made from out side State. (2) Sale records: The dealer shall have to maintain the following sales records: (i) All sales invoice in serial number. (ii) Separate records of tax invoice and retail invoices. (iii) Copies of all credit and debit notes issued in chronologic order. (iv) Sales records showing separately sales, made at different tax rates. (v) Detail of the amount of tax charged on each sale. (3) Complete details of VAT Acount: The dealer shall have to maintain the following records for such purpose: (i) Records of input tax (ii) Records of output tax (iii) The net tax payable in each period or (iv) The excess carried forward. (4) Other records to be maintained: (a) Record of orders received and delivery challans, wherever applicable (b) Bank records (c) Cash book, day book and ledger (d) Annual accounts including Trading, Profit and loss account and Balance Sheet (e) Tax audit report. 27.6 Return of VAT A registered dealers shall be required to file a return alongwith the requisite details such as output tax liability, value of input tax credit, payment of VAT. Simplified return of VAT are to filed monthly or quarterly by the dealer as specified by each State. Returns shall be accompanied with the challans evidencing payment of tax. Every return furnished by dealers will be scrutinized expeditiously within prescribed time limit from the date of filing the return. If any technical mistake is detected on scrutiny, the dealer will be required to pay the deficit appropriately. 27.7 Self-assessment by dealers Procedure of Self-Assessment of VAT Liability The basic simplification in VAT is that VAT liability will be self-assessed by the dealers themselves in terms of submission of returns upon setting off the tax

724

Systematic Approach to VAT

Chap. 27

credit. Return forms as well as other procedures are simple in all States. There will not longer be compulsory assessment at the end of each year as is existing now. If no specific notice is issued proposing departmental audit of the books of accounts of the dealer within the time limit specified in the Act the dealer will be deemed to have been self-assessed on the basis of returns submitted by him. 27.8 Audit under VAT has been made compulsory by various States There shall no longer be compulsory assessment at the end of each year. Correctness of self-assessment will be checked through a system of Departmental Audit. A certain percentage of the dealers will be taken up for audit every year on a scientific basis. If, however, evasion is detected on audit, the concerned dealer may be taken up for audit for previous periods. This Audit Wing will remain delinked from tax collection wing to remove any bias. The audit team will conduct its work in a time bound manner and audit will be completed within six months. The audit report will be transparently sent to the dealer also. Simultaneously, a cross-checking, computerized system is being worked out on the basis of coordination between the tax authorities of the State Government and the authorities of Central Excise and Income Tax to compare constantly the tax returns and set-off documents of VAT system of the States and those of Central Excise and Income Tax. This comprehensive cross-checking system will help reduce tax evasion and also lead to significant growth of tax revenue. 27.9 No declaration form required There is no requirement of any declaration form to be obtained/issued as invoice will be raised for each sale and VAT shall be levied. 27.10 Most of the commodities covered under VAT There is compulsory coverage of all goods under VAT as only few commodities have been exempted from VAT.

ILLUSTRATIONS ON VALUE ADDED TAX


Illustration 27.1 R is a trader in Delhi who purchased goods from 'X' of Kolkata for `6,00,000 and paid CST @ 2% on such purchases. He sold the entire goods purchased from X to S of Delhi for `7,50,000 and charged VAT @ 12.5%. Further, R purchased goods 'Y' in Delhi for `8,00,000 and paid VAT @ 12.5%. The whole of goods 'Y' was sold for `10,00,000 to a registered dealer in Punjab and & R charged central sale tax @ 2%. Compute the VAT payable by R. Solution Computation of VAT payable by R ` ` Purchases from Kokata Input tax

Chap. 27
Purchase price Add: CST @ 2% Cost to R Sale price VAT @ 12.5%

General requirement for VAT System


6,00,000 12,000 6,12,000 7,50,000 93,750 8,43,750

725
credit available Nil

Net VAT payable by R 93,750 Nil = `93,750 Purchases for Delhi Purchase price Add: VAT @ 12.5%

Sale price + CST @ 2%

CST payable Total VAT credit available Nil + 1,00,000 VAT payable on sale in Delhi

8,00,000 1,00,000 9,00,000 10,00,000 20,000 10,20,000 20,000 1,00,000 93,750

Input tax credit available 1,00,000

CST payable `20,000 6,250 = `13,750. Illustration 27.2 R submit you the following information. Purchase of raw-material from X inclusive of VAT @ 4% 6,24,000 Manufacturing and other expenses 3,00,000 Profit 25% on cost The entire manufactured goods are sold to S a dealer by charging Central Excise Duty @ 10% plus education cess & SHEC @ 3%. The rate of VAT for the manufactured goods is 12.5%. Dealers sells the goods to T by charging profit @ 20% on cost. Compute the excise duty and VAT payable by R. How much input tax credit shall be allowed to S and what shall be VAT payable by S. Solution Sale price of the manufactured goods Cost of raw material purchased ` Input VAT credit available `6,24,000 100/104 6,00,000 24,000 Add: Manufacturing expenses 3,00,000 9,00,000 Add: Profit @ 25% on cost 2,25,000 Sale price 11,25,000 Bill to be raised by R for sale made to S. Sale price as computed above 11,25,000 Add: Excise duty @ 10.3% 1,15,875 VAT payable `1,55,109 Total value of sale price 12,40,875 Add: VAT @ 12.5% (rounded off) 1,55,109 13,95,984 Net VAT payable by R `1,55,109 24,000 = 1,31,109 Cost of goods in the hand of S `13,95,984 1,55,109 (VAT included in invoice) 12,40,875 Input credit available

726
Add: Profit 20% on cost

Systematic Approach to VAT

Chap. 27

2,48,175 `1,55,109 14,89,050 Add: VAT @ 12.5% (rounded off) 1,86,131 VAT payable `1,86,131 16,75,181 Net VAT payable by S `1,86,131 1,55,109 = `31,022. Note.Cenvat credit of excise duty is not allowed to S as he is not a manufacturer but a trader. Illustration 27.3 R is a manufacturer at Delhi and has purchased raw-material A from 'X' a manufacturer at Delhi for `3,00,000 who charged excise duty @ 10.3% and VAT @ 4%. He also purchased another raw material 'B' from Y of Mumbai for `2,00,000 who charged excise duty @ 10.3% and CST @ 2%. The manufacturing and other expenses incurred by R were `2,00,000 and profit included were `60,000. The final product was sold to S a trader in Delhi. Excise duty charged was 10.3% and VAT charged was 12.5%. S after incurring expenditure of `50,000 and adding profit @ 25% on cost sold the goods to T. Compute the excise duty payable by R and VAT payable by R & S. Solution (1) The invoice raised by X to R Sale price of raw-material 'A' 3,00,000 Add: Excise duty @ 10.3% 30,900 3,30,900 Add: VAT @ 4% 13,236 Total sale price 3,44,136 (2) Invoice raised by Y to R Sale price of raw-material 'B' 2,00,000 Add: Excise duty @ 10.3% 20,600 2,20,600 Add: CST @ 2% 4,412 Total sale price 2,25,012 Sale price of goods sold by R to S Cenvat credit Input VAT Cost of raw-material 'A' excluding excise duty & VAT 3,00,000 30,900 13,236 Cost of raw-material 'B' including CST but including excise duty 2,04,412 20,600 Nil Total cost of raw material 5,04,412 51,500 13,236 Manufacturing and other expenses 2,00,000 Profit 60,000 7,64,412 Excise duty 10.3% (rounded off) 78,734 8,43,146 Add: 12.5% (rounded off) 1,05,393 9,48,539 Central Excise Payable after cenvat credit of excise duty `78,734 51,500 = 27,234 VAT payable after taking input tax credit

Chap. 27
`1,05,393 13,236 = 92,157 Cost of goods to S Total cost Less: Input VAT credit Balance Sale price of goods sold to T

General requirement for VAT System

727

9,48,539 1,05,393 8,43,146 Cenvat credit available ` ` 8,43,146 Nil 50,000 8,93,146 2,23,287 11,16,433 1,39,554 12,55,987 input VAT available ` 1,05,393

Cost of goods Add: Expenses incurred Add: Profit @ 25% on cost (rounded off) Add: VAT 12.5%

Net VAT payable `1,39,554 1,05,393 = `34,161 by S. S will not be entitled to Cenvat credit of S as he is not the manufacturer. Illustration 27.4 R a dealer at Delhi purchased goods from dealer S of Delhi for `13,50,000 including VAT @ 12.5%. R earns a profit @ 25% on the cost and sold the same goods to a retailer T. Compute the amount of VAT payable by R. Solution Purchase price of goods including VAT 13,50,000 Input credit available Less: Input tax `13,50,000 12.5/112.5 1,50,000 1,50,000 Cost to R 12,00,000 Add: Profit 25% on `12,00,000 3,00,000 Sale price 15,00,000 VAT @ 12.5% output tax 1,87,500 16,87,500 Net VAT payable `1,87,500 15,000 = `3,75,000. Illustration 27.5 R, a manufacturer in Delhi, purchased raw-material 'A' from Haryana for `6,00,000 and paid CST @ 2%. He purchased raw-material 'B' from Delhi for `8,00,000 and paid VAT @ 4%. He incurred `2,00,000 as manufacturing and other expenses and earned a profit of `1,00,000. 60% of the goods were sold in Delhi and VAT charged was 12.5% and remaining 40% of the goods were sold to Dealer in Maharastra and CST was charged @ 2%. Compute the VAT and CST payable. Solution Cost of raw-material 'A' purchased from Haryana 6,12,000 No input credit including CST @ 2% for CST Cost of raw-material 'B' purchased from Delhi 8,00,000 Input credit available `32,000 Total cost of raw-material 14,12,000 Manufacturing other expenses 2,00,000 Profit 1,00,000 Total 17,12,000

728

Systematic Approach to VAT


10,27,200 1,28,400 11,55,600 6,84,800 13,696 6,98,496 ` 96,400 13,696

Chap. 27

Value of 60% of goods sold in Delhi 17,12,000 60/100 Add: VAT charged @ 12.5%

Value of 40% of goods sold to dealer in Maharashtra 17,12,000 40/100 Add: CST @ 2%

Net VAT payable `1,28,400 32,000 (input tax credit) CST payable

Illustration 27.6 From the following information submitted by R, a registered dealer at Delhi, compute the tax liability for the year ended 31.3.2011. Purchase of raw-material with in Delhi inclusive of VAT @ 4% 8,32,000 Inter State purchases of raw-materials inclusive of CST @ 2% 3,12,000 Import of raw-material from Germany inclusive of customs duty of `50,000 5,50,000 Capital goods purchased on 1.4.2010 inclusive of VAT @ 12.5% (input credit to be spread over 24 months) 2,25,000 Manufacturing and other expenses 1,60,000 Sale of taxable goods within State inclusive of VAT levy @ 4% 16,64,000 Sale of taxable goods outside the State inclusive of CST @ 2% 6,12,000 20% of the raw-material purchased from Delhi was still in stock as on 31.3.2011. Solution (1) Sale price of taxable goods within State inclusive of VAT @ 4% 16,64,000 Less: VAT payable 16,64,000 4/104 64,000 (2) Sale price of taxable goods sold outside the State 6,12,000 Less: CST payable `6,12,000 2/102 12,000 Input credit available on purchase of raw-material Purchase of raw-material from Delhi 32,000 Purchase of raw-material from other States. CST paid is not available for input VAT credit Nil Purchase of raw-material from Germany Nil Custom duty paid is not available for input VAT credit Input VAT credit on capital goods `2,25,000 12.5/112.5 = 25,000 12 months/24 months 12,500 44,500 ` VAT payable 64,000 Less: Input credit available 44,500 Net VAT payable 19,500 CST payable 12,000 Input VAT credit can be taken immediately even though the raw-material is lying in stock-in-trade.

MULTIPLE CHOICE QUESTIONS


1 Tax is levied under VAT at (A) Last stage of sale (B) Multistage (C) First stage of sale (D) First and last stage of sale

Chap. 27

General requirement for VAT System

729

Write True or False 1 Input credit under VAT is available on account of Central Sales Tax paid on the purchases 2 Input credit is available on account of import duty paid on goods purchased from a country out side India 3 Input credit is payable on account of VAT paid on capital goods 4 Input credit is available only if the purchaser has obtained proper tax in voice 5 VAT avoids cascading effect 6 Vat is easy to administer and transparent Answers to multiple choice questions 1. (B) Answers to True or False 1 (F) 3 (T) 5 (T) 2 (F) 4 (T) 6 (T)

PRACTICAL QUESTIONS*
1. Compute the invoice value to be charged and amount of tax payable under VAT by a dealer who had purchased goods for `1,20,000 and after adding for expenses of `10,000 and of profit `15,000 had sold out the same. The rate of VAT on purchases and sales is 12.5%. Ans.: Invoice value `1,63,125, VAT payable `3,125. [See problem 28.1 of Practical Approach] 2. Manufacturer A sold product X to B of Delhi @ `1000 per unit. He has charged CST @ 2% on the said product and paid `80 as freight. B of Delhi sold goods to C of Delhi @ `1250 per unit and charged VAT @ 12.5%. C of Delhi sold goods to D, a consumer @ `1500 per unit and charged VAT @ 12.5%. Ans.: VAT payable B `156.25, C `31.25. [See problem 28.2 of Practical Approach] 3. Determine how much input credit shall be available to the dealer 'R' in Delhi in respect of the following purchases: (1) Goods purchased from Mumbai `3,06,000 which includes Central Sales Tax (CST) @ 2%. (2) Goods purchased from a dealer in Delhi 'A' `3,60,000. VAT charged 12.5% i.e. `45,000. Total value of purchase invoice `4,05,000. (3) Goods purchased from unregistered dealer `40,000. (4) Goods purchased from a dealer 'Y' under composition scheme `80,000. Y has paid 1% as tax under composition scheme. (5) Purchases from dealer Z in Delhi for `2,80,000. VAT charged @ 5% `14,000. Total value of purchase `2,94,000. (6) Purchases from dealer B in Delhi `75,000. VAT is not separately charged in the invoice. (7) Purchase of capital goods `10,12,500. `9,00,000 is price of capital goods and `1,12,500 is VAT amount separately charged. (8) Goods purchased `1,05,000 which includes 5% VAT which is separately shown. Such goods have been utilized in the manufacture of exempted goods. (9) Value of goods imported from Germany `9,00,000. Ans.: `1,71,500. [See problem 27.8 of Practical Approach] 4. Compute the VAT amount payable by R, who purchased goods from a manufacturer on payment of `6,30,000 (including VAT) and earned 20% profit on sale price. VAT rate both on

For solutions to these Practical Questions, refer Bharat's Practical Approach to Income Tax, Wealth Tax, CST, Service Tax & VAT (Problems and Solutions), 2011 edition, by the same authors.

730

Systematic Approach to VAT

Chap. 27

purchases and sales is 5%. Ans.: `7,500. [See problem 27.9 of Practical Approach] 5. R. a manufacturer, sells goods to S, a distributor for `2,40,000 (exclusive of VAT) S sells goods to T a wholesale dealer for `3,00,000. The wholesale dealer sells the goods to a retailer for `4,00,000, who ultimately sells to the consumers for `5,00,000. Compute the Tax Liability, input credit availed and tax payable by the manufacturer, distributor, wholesale dealer and retailer under invoice method assuming VAT rate @ 12.5%. Ans.: R `30,000, S `7,500, K `12,500, Retailer `12,500. [See problem 27.10 of Practical Approach] 6. (a) R, a dealer in Delhi dealing in consumer goods, submits the following information pertaining to the Month of March, 2011: (i) Exempt goods 'X' purchased for `3,00,000 and sold for `3,60,000. (ii) Goods 'Y' purchased for `5,62,500 (including VAT) and sold at a margin of 15% profit on purchases (VAT rate 12.5%). (iii) Goods 'Z' purchased for `2,00,000 (excluding VAT) and sold for `2,80,000 (VAT rate 5%). (iv) His unutilized balance in VAT input credit on 1.3.2011 was `3,400. Compute the turnover, Input VAT, Output VAT and Net VAT payable by R. Ans.: Turnover `12,15,000, Input VAT `75,900, Output VAT `85,875, Net VAT payable `9,975. [See problem 27.11 of Practical Approach]

You might also like